Формула расчета сопротивления при параллельном соединении резисторов: Формула расчета сопротивления при параллельном соединении резистора

Содержание

Формула расчета сопротивления при параллельном соединении резистора

Параллельное соединение резисторов — одно из двух видов электрических соединений, когда оба вывода одного резистора соединены с соответствующими выводами другого резистора или резисторов. Зачастую резисторы соединяют последовательно или параллельно для того, чтобы создать более сложные электронные схемы.

Схема параллельного соединения резисторов показан на рисунке ниже. При параллельном соединении резисторов, напряжение на всех резисторах будет одинаковым, а протекающий через них ток будет пропорционален их сопротивлению:

Формула параллельного соединения резисторов

Общее сопротивление нескольких резисторов соединенных параллельно определяется по следующей формуле:

Ток, протекающий через отдельно взятый резистор, согласно закону Ома, можно найти по формуле:

При разработке устройства, возникла необходимость установить резистор с сопротивлением 8 Ом.

Если мы просмотрим весь номинальный ряд стандартных значений резисторов, то мы увидим, что резистора с сопротивлением в 8 Ом в нем нет.

Выходом из данной ситуации будет использование двух параллельно соединенных резисторов. Эквивалентное значение сопротивления для двух резисторов соединенных параллельно рассчитывается следующим образом:

Данное уравнение показывает, что если R1 равен R2, то сопротивление R составляет половину сопротивления одного из двух резисторов. При R = 8 Ом, R1 и R2 должны, следовательно, иметь значение 2 × 8 = 16 Ом.
Теперь проведем проверку, рассчитав общее сопротивление двух резисторов:

Таким образом, мы получили необходимое сопротивление 8 Ом, соединив параллельно два резистора по 16 Ом.

Пример расчета №2

Найти общее сопротивление  R из трех параллельно соединенных резисторов:

Общее сопротивление R рассчитывается по формуле:

Этот метод расчета может быть использованы для расчета любого количества отдельных сопротивлений соединенных параллельно.

Один важный момент, который необходимо запомнить при расчете параллельно соединенных резисторов – это то, что общее сопротивление всегда будет меньше, чем значение наименьшего сопротивления в этой комбинации.

Как рассчитать сложные схемы соединения резисторов

Более сложные соединения резисторов могут быть рассчитаны путем систематической группировки резисторов. На рисунке ниже необходимо посчитать общее сопротивление цепи, состоящей из трех резисторов:

Резисторы R2 и R3 соединены последовательно (группа 2). Они в свою очередь соединены параллельно с резистором R1 (группа 1).

Последовательное соединение резисторов группы 2 вычисляется как сумма сопротивлений R2 и R3:

В результате мы упрощаем схему в виде двух параллельных резисторов. Теперь общее сопротивление всей схемы можно посчитать следующим образом:

Расчет более сложных соединений резисторов можно выполнить используя законы Кирхгофа.

Ток, протекающий в цепи параллельно соединенных резисторах

Общий ток I протекающий в цепи параллельных резисторов равняется сумме отдельных токов, протекающих во всех параллельных ветвях, причем ток в отдельно взятой ветви не обязательно должен быть равен току в соседних ветвях.

Несмотря на параллельное соединение, к каждому резистору приложено одно и то же напряжение. А поскольку величина сопротивлений в параллельной цепи может быть разной, то и величина протекающего тока через каждый резистор тоже будет отличаться (по определению закона Ома).

Рассмотрим это на примере двух параллельно соединенных резисторов. Ток, который течет через каждый из резисторов ( I1 и I2 ) будет отличаться друг от друга поскольку сопротивления резисторов R1 и R2 не равны.
Однако мы знаем, что ток, который поступает в цепь в точке «А» должен выйти из цепи в точке «B» .

Первое правило Кирхгофа гласит: «Общий ток, выходящий из цепи равен току входящий в цепь».

  • Таким образом, протекающий общий ток в цепи  можно определить как:
  • I = I1 + I2
  • Затем с помощью закона Ома можно вычислить ток, который протекает через каждый резистор:
  • Ток, протекающий в R1 = U ÷ R1 = 12 ÷ 22 кОм = 0,545 мА
  • Ток, протекающий в R 2 = U ÷ R2 = 12 ÷ 47 кОм = 0,255 мА
  • Таким образом, общий ток будет равен:
  • I = 0,545 мА + 0,255 мА = 0,8 мА
  • Это также можно проверить, используя закон Ома:
  • I = U ÷ R = 12 В ÷ 15 кОм = 0,8 мА (то же самое)
  • где 15кОм — это общее сопротивление двух параллельно соединенных резисторов (22 кОм и 47 кОм)
  • И в завершении хочется отметить, что большинство современных резисторов маркируются цветными полосками и назначение ее можно узнать здесь.

Параллельное соединение резисторов — онлайн калькулятор

Чтобы быстро вычислить общее сопротивление двух и более резисторов, соединенных параллельно, вы можете воспользоваться следующим онлайн калькулятором:

Подведем итог

Когда два или более резистора соединены так, что оба вывода одного резистора соединены с соответствующими выводами другого резистора или резисторов, то говорят, что они соединены между собой параллельно. Напряжение на каждом резисторе внутри параллельной комбинации одинаковое, но токи, протекающие через них, могут отличаться друг от друга, в зависимости от величины сопротивлений каждого резистора.

Эквивалентное или полное сопротивление параллельной комбинации всегда будет меньше минимального сопротивления резистора, входящего в параллельное соединение.

Источник: http://www.joyta.ru/7362-parallelnoe-soedinenie-rezistorov/

Последовательное и параллельное соединение резисторов

Последовательное соединение – это соединение двух или более резисторов в форме цепи, в которой каждый отдельный резистор соединяется с другим отдельным резистором только в одной точке.

Общее сопротивление Rобщ

При таком соединении, через все резисторы проходит один и тот же электрический ток. Чем больше элементов на данном участке электрической цепи, тем «труднее» току протекать через него. Следовательно, при последовательном соединении резисторов их общее сопротивление увеличивается, и оно равно сумме всех сопротивлений.

Напряжение при последовательном соединении

Напряжение при последовательном соединении распределяется на каждый резистор согласно закону Ома:

Т.е чем большее сопротивление резистора, тем большее напряжение на него падает.

Параллельное соединение резисторов

Параллельное соединение – это соединение, при котором резисторы соединяются между собой обоими контактами. В результате к одной точке (электрическому узлу) может быть присоединено несколько резисторов.

Общее сопротивление Rобщ

При таком соединении, через каждый резистор потечет отдельный ток. Сила данного тока будет обратно пропорциональна сопротивлению резистора. В результате общая проводимость такого участка электрической цепи увеличивается, а общее сопротивление в свою очередь уменьшается.

Таким образом, при параллельном подсоединении резисторов с разным сопротивлением, общее сопротивление будет всегда меньше значения самого маленького отдельного резистора.

Формула общей проводимости при параллельном соединении резисторов:

Формула эквивалентного общего сопротивления при параллельном соединении резисторов:

Для двух одинаковых резисторов общее сопротивление будет равно половине одного отдельного резистора:

Соответственно, для n одинаковых резисторов общее сопротивление будет равно значению одного резистора, разделенного на n.

Напряжение при параллельном соединении

Напряжение между точками A и B является как общим напряжением для всего участка цепи, так и напряжением, падающим на каждый резистор в отдельности. Поэтому при параллельном соединении на все резисторы упадет одинаковое напряжение.

Электрический ток при параллельном соединении

Через каждый резистор течет ток, сила которого обратно пропорциональна сопротивлению резистора. Для того чтобы узнать какой ток течет через определенный резистор, можно воспользоваться законом Ома:

Смешанное соединение резисторов

Смешанным соединением называют участок цепи, где часть резисторов соединяются между собой последовательно, а часть параллельно. В свою очередь, смешанное соединение бывает последовательного и параллельного типов.

Общее сопротивление Rобщ

Для того чтобы посчитать общее сопротивление смешанного соединения:

  • Цепь разбивают на участки с только пареллельным или только последовательным соединением.
  • Вычисляют общее сопротивление для каждого отдельного участка.
  • Вычисляют общее сопротивление для всей цепи смешанного соединения.

Так это будет выглядеть для схемы 1:

Также существует более быстрый способ расчета общего сопротивления для смешанного соединения. Можно, в соответствии схеме, сразу записывать формулу следующим образом:

  • Если резисторы соединяются последоватеьно — складывать.
  • Если резисторы соединяются параллельно — использовать условное обозначение «||».
  • Подставлять формулу для параллельного соединения где стоит символ «||».

Так это будет выглядеть для схемы 1:

После подстановки формулы параллельного соединения вместо «||»:

Источник: http://hightolow.ru/resistor3.php

Параллельное соединение сопротивлений в электрической цепи. Параллельное соединение конденсаторов и катушек

Параллельное соединение электрических элементов (проводников, сопротивлений, емкостей, индуктивностей) — это такое соединение, при котором подключенные элементы цепи имеют два общих узла подключения.

Другое определение: сопротивления подключены параллельно, если они подключены одно и той же паре узлов. 

Графическое обозначение схемы параллельного соеднинения

На приведенном рисунке показана схема параллельное подключения сопротивлений R1, R2, R3, R4. Из схемы видно, что все эти четыре сопротивления имеют две общие точки (узла подключения). 

В электротехнике принято, но не строго требуется, рисовать провода горизонтально и вертикально. Поэтому эту же схему можно изобразить, как на рисунке ниже. Это тоже параллельное соединение тех же самых сопротивлений.

Формула для расчета параллельного соединения сопротивлений

При параллельном соединении обратная величина от эквивалентного сопротивления равна сумме обратных величин всех параллельно подключенных сопротивлений. Эквивалентная проводимость равна сумме всех параллельно подключенных проводимостей электрической схемы.

Для приведенной выше схемы эквивалентное сопротивление можно рассчитать по формуле:

В частном случае при подключении параллельно двух сопротивлений:

Эквивалентное сопротивление цепи определяется по формуле:

 В случае подключения «n» одинаковых сопротивлений, эквивалентное сопротивление можно рассчитать по частной формуле:

Формулы для частного рассчета вытекают из основной формулы.  

Формула для расчета параллельного соединения емкостей (конденсаторов)

При параллельном подключении емкостей (конденсаторов) эквивалентная емкость равна сумме параллельно подключенных емкостей:

 

Формула для расчета параллельного соединения индуктивностей

  • При параллельном подключении индуктивностей, эквивалентная индуктивность рассчитывается так же, как и эквивалентное сопротивление при параллельном соединении: 
  •  
  • Необходимо обратить внимание, что в формуле не учтены взаимные индуктивности.

Пример свертывания параллельного сопротивления  

Для участка электрической цепи необходимо найти параллельное соединение сопротивлений выполнить их преобразование до одного.

Из схемы видно, что параллельно подключены только R2 и R4. R3 не параллельно, т.к. одним концом оно подключено к источнику ЭДС E1. R1 — одним концом подключено к R5, а не к узлу. R5 — одним концом подключено к R1, а не к узлу. Можно так же говорить, что последовательное соединение сопротивлений R1 и R5 подключено параллельно с R2 и R4.

Рассчитать эквивалентное сопротивлений R14 можно по формуле для двух сопротивлений.

Ток при параллельном соединении

При параллельном соединении сопротивлений ток через каждое сопротивление в общем случае разный. Величина тока обратно пропорциональна величине сопротивления.

Напряжение при параллельном соединении 

При параллельном соединении разность потенциалов между узлами, объединяющими элементы цепи, одинакова для всех элементов.

Применение параллельного соединения

1. В промышленности изготавливаются сопротивления определенных величин. Иногда необходимо получить значение сопротивления вне данных рядов. Для этого можно подключить несколько сопротивлений параллельно. Эквивалентное сопротивление всегда будет меньше самого большого номинала сопротивления.

2. Делитель токов.

Источник: https://kurstoe.ru/osnovnie-svedeniya/preobrazovanie-tcepej/parallelnoe-soedinenie.html

Последовательное и параллельное соединение резисторов

Последовательное и параллельное соединение резисторов в схемах являются самыми распространенными, также — это база для расчета более сложных схем.

Последовательное подключение

Начнем с последовательного соединения. По этой схеме каждый резистор подключается с другим только в одной точке, их может быть в цепи 2, 3 и больше.

Рис. Последовательное подключение.

Обозначение:

Обозначим сопротивления: R1, R2, R3 и напряжение источника в цепи Uц. При подключении источника питания в ней начнет протекать ток Iц. В цепи с последовательным соединением ток протекает по всем резисторам один за другим.

Поскольку ток течет через все резисторы их сопротивления и ток суммируется, Iц = I1+I2+I3, Rц = R1 +R2 + R3, чем больше отдельно взятое сопротивление, тем тяжелее электронам преодолевать участок цепи. Мощность резисторов при последовательном и параллельном соединении рассчитывается по разным формулам.

В последовательных цепях — складываем, в параллельных — это обратно пропорциональная величина.

Параллельное соединение

Рис. Параллельное подключение.

Данный вид подключения характерен тем, что все элементы цепи соединяется выводами в одной точке друг другу, т. е. точка входа и выхода всех нагрузок сходятся в одну точку (или еще одно обозначение на схемах — //). Электроток, двигаясь по проводнику, дойдя до общего соединения делится на количество имеющихся веток. Если представить движение воды в трубе, то можно сказать, что вода двигающиеся по одной трубе, равномерно перетекает в несколько отводов, подсоединенных к ней. В нашем случае заряженные электроны, двигающиеся по проводнику, также растекаются на количества предложенных веток в узле. Более наглядно это можно представить в виде формул: 1. Каждый вид соединения находится под одинаковым напряжением: U = U1 = U2; 2. Суммарная сила тока равняется суммарному значению тока каждого участка I = I1 + I2; 3. Сопротивление цепи равно сумме величина обратных сопротивлению участка: 1/R = 1/R1 + 17R2 + . . . + 1/Rn; 4. Сила тока пропорциональна сопротивлению каждого участка I1/I2=R2/R1.

Далее рассмотрим схему как работает не только последовательное параллельное, но и смешанное соединение резисторов.

Смешанное подключение

Рис. Смешанное подключение резисторов

В электрических схемах используются не только типовые схемы, но и смешанное, созданное из критерий определенных требований. Чаще всего в схемах встречается третий вариант, представляющий набор из элементарных типов схем. В смешанных участках учитываются не только элементы, но и направления движения тока. При вычислении мощности резисторов смешанного подключения используются формулы для параллельного и последовательного соединения резисторов, формула также является составной.

Основные законы электротехники, наиболее часто используемые для расчетов

Рассмотрим основные законы электротехники и свойства последовательного и параллельного соединения резисторов для участка цепи

Закон Ома

Напряжение находится по закону Ома по формуле I=U/R — чем больше сопротивление, тем меньше ток. Напряжение можно найти из этой же формулы. U=R*I, ток умножается на сопротивление. Запишем эту формулу для каждого участка U1=R1· I1, Un=Rn · In.

Законы Кирхгофа

Первый закон

Ещё один очень важный закон — это закон Кирхгофа. Для участка цепи постоянного тока их два.

Рис. иллюстрация к пояснению действия первого закона Кирхгофа.

Первый закон имеет формулировку: Сумма всех токов, входящих в узел и выходящих из него равна нулю. Если посмотреть на схему, I1 — это ток, который заходит в узел, I2 и I3 — это электроны, которые вытекают из него. Применяя формулировку первого закона можно записать формулу по-другому:

I1-I2+I3=0. В этой формуле знаки плюс имеют значения, которые прибывают в узел, минус, который отходит от него.

Второй закон Кирхгофа

Рис. иллюстрация к пояснению действия второго закона Кирхгофа.

Если к цепи с включенными сопротивлениями подключен один источник ЭДС (батарея питания) тогда всё понятно, можно обойтись законом Ома. А, если, источников несколько и схема с различным схемным расположением элементов, тогда вступает в силу второй закон, который гласит: сумма токов всех источников питания для замкнутого контура, равна сумме падений напряжения на всех сопротивлениях участка в этом контуре. E1- Е2 = — UR1 — UR2 или E1 = Е2 — UR1 — UR2.

Параллельное и последовательное соединение резисторов,  решение задач

Алгоритм расчёта смешанных подключений находится в тех же правилах, что и в элементарных схемах расчета последовательного и параллельного соединения резисторов. Ничего нового нет: нужно правильно разбить предложенную схему на пригодные для расчета участки. Участки, с элементами, подключены поочередно либо параллельно.

Рис. Порядок замещения при расчете сложных позиций более простыми.

Для решения задачи на последовательное и параллельное соединение резисторов необходимо правильно оценить цепи элементов. Рассмотрим схему №1 на рис.

На схеме присутствует параллельная и последовательная часть соединения элементов. Для расчета очень важно аккуратно, шаг за шагом упрощать цепи и не брать сразу всю схему (рис.1).

Как же правильно определить параллельное и последовательное соединение резисторов?

Для примера расчета возьмем резисторы R3, R4, которые подключены параллельно. Эквивалентный резистор этих элементов, будет равенRэ. = 1/R34 =1/R3 + 1/R4, после преобразования формулы и приведения к одному знаменателю получим R34 = R3 · R4 / (R3 + R4). Э. = 1/3+1/4 /(3+4) =1,7 Ом.

Далее видно, что приведённая эквивалентное R эк и R6 соединены последовательно, чтобы узнать сопротивление их необходимо сложить, тогда общее сопротивление будет равно R346 = R34 + R6, тогда Rэк346 = 1,7 + 6 = 7, 7 Ом. Заменяем на схеме одним общим элементом, теперь, позиция упрощается еще больше (рис 3).

Теперь образовалась ситуация — включение трех элементов в //. Как вычисляется такое соединение нам уже известно, 1/ R23465 = 1/ R2 +1/R346 + 1/R5 после вычисления правой части получаем 0,82 Ом. После окончательного вычисления получаем R23465 = 2,1 Ом. Здесь следует обратить внимание, что общее сопротивление получилось меньше самого меньшего из трех.

Заменяем эти сопротивление одним эквивалентным R23465. В конечном итоге все выглядит уже намного проще. Rц = Rэк + R1+ R2. R об. = R ц = 1,21 +7+1 =9,21 Ом. Из приведенного алгоритма расчёта видно, как из сложной схемы путем простого математического вычисления и применения правил сокращения резисторов участок становится простой и понятной.

Схема с подключением сопротивлений «треугольником»

Рис. Расчетная схема соединения резисторов в треугольник.

Иногда некоторые затруднения возникают при разборе схемы соединения в треугольник.

Рассмотрим на примере рисунка расчет резисторов по этому подключению. Из схемы видно, что R1 и R2 соединены последовательно Rэ12 будет соединяться R3 последовательно.

Затем Rэ123 соединяется с сопротивлением R4, R5 в последовательную цепь. Затем все это объединяется с Rэ в //.

Проведем несложные вычисления учитывая, что R1, R2, R4, R5 равняется 1 Ом. R3, R7 — 2 Ом.

RЭ1,2 = R1+R2 = 1+1=2 Ом.

Вычисляем параллельное подключение: Rэ 12 с R3. Rэ1,3 = (Rэ12*R3) /(Rэ12+R3) = (2*2) /(2+2) = 1Ом.

Далее мы видим последовательное: RЭ123 + R4 + R5 = 1+1+1 = 3 Ом. И последнее — Rэ123 4 5 с R6 — параллельное.

Общее сопротивление цепи Rц = Rоб = (RЭ1,2,3,4,5 *R6) /(RЭ1,2,3,4,5+R6) = (3 * 2) / (3+2) = 1,2 Ом. Как видно, что расчет подобного варианта также не сложный.

Расчет последовательного и параллельного подключения резисторов онлайн

Подсчитать значение мощность и сопротивлений подставляя их в формулы можно только в учебных целях, или, когда объемы не очень большие.

Наиболее практичный вариант расчета является онлайн калькуляторы, которые расположены на многочисленных интернет ресурсах.

Для расчёта любой сложности нужно правильно определить тип соединения резисторов последовательное или параллельное и внести данные для расчета в поля калькулятора.

Также такая форма расчета подойдет и для проверки результатов решения учебных задач.

Последовательное и параллельное соединение резисторов и конденсаторов

Электрические цепи состоят не только из резисторов, в них применяется большое количество различных деталей, например, конденсатор, которые подключаются в последовательное, // и смешанное соединение.

Рис. Замещения последовательно включенных элементов.

Определение этому элементу можно дать следующее: Конденсатор — это совокупность проводящих тел служащий для накопления электрического заряда. Элементарный конденсатор имеет две пластины, форма этих пластин может быть различной: сферической, круглой, цилиндрической, прямоугольной — по форме пластин разделяется и тип конденсатора.

Важное свойство. Одно из важных свойств конденсатора: если заряжается одна пластина конденсатора, то благодаря явлению электростатической индукции заряжается и вторая половина, но с противоположным знаком.

Устройство конденсатора

Плоский конденсатор состоит из двух плоских пластин отстоящих друг от друга на маленькое расстояние. У конденсатора к двум пластинам припаивается вывод всего их получается два.

Типовые схемы подключения конденсаторов

Рассмотрим различные виды подключения конденсатора.

Последовательное

Первый вид — это последовательное соединение. Предположим, что емкость этих конденсаторов будут равны.

Тогда заряды также будут равны: q1=q2=q3, как и в примере с резисторами,  сложный тип позиций с конденсатором можно упростить, заменив несколько элементов одним.

У элементов соединенных друг за другом, общая емкость будет обратно пропорциональная всем имеющимся элементам. То есть: Rэк будет равняться 1/С1 + 1/С2 +…. 1/Сn/

Напряжение складывается,  U эк = U1 + U2+ … Un.

Параллельное

Второй тип подключения конденсаторов — это соединение в паралель

Рис. Схема замещения элементов, включенных в параллель.

  • Соответственно эти конденсаторов обозначены C1, C2, … Cn заряды: Q1, Q2, … Qn и напряжение: U1, U2, … Un.
  • У элементов в // емкость складывается Сэ = C1 + C2 + … C n. Напряжение Un на каждом конденсаторе будет равно напряжению на эквивалентном
  • Uэ = U1 = U2 =… = Un — это особенность параллельного подсоединения всех элементов цепи.
  • Емкость будет складываться из суммы отдельных элементов Сэ =С1 + С2 + … Сп.

Рис. Расчетные позиции элементов при различном включении.

Простая позиция, которая не требует преобразования №1 — последовательное подключение. По известной формуле для этих поз. запишем 1/Сэ = 1/С1 +1/С2 +1/С3,  подставив формулу значения, которые даны в условии задачи, получим 1/Сэ = 1/С1 +1/С2 +1/С3 = 59 мФ.

Не требует преобразования и 2 схема: емкость общего конденсатора будет равняться сумме конденсаторов которые включены в параллельной цепи: Сэ =С1 +С2 +С3 Сэ = 100 + 200 + 500 = 800 мФ.

Рассмотрев рис. №3 видно, что пара конденсаторов включена параллельно и один последовательно. Алгоритм преобразования таких цепей мы уже рассматривали, поэтому: сразу же находим емкость конденсатора Сэ соединения: Сэ = С1+С2 = 200+500 = 700 мФ.

Теперь находим общие эквивалентную емкость элементов с последовательным подключением 1/Сэ = 1/С2,3 +1/ С1 = 89 мф. Практическая задача решена.

Источник: http://themechanic.ru/posledovatelnoe-i-parallelnoe-soedinenie-rezistorov/

Соединение резисторов

Радиоэлектроника для начинающих

О том, как соединять конденсаторы и рассчитывать их общую ёмкость уже рассказывалось на страницах сайта. А как соединять резисторы и посчитать их общее сопротивление? Именно об этом и будет рассказано в этой статье.

Резисторы есть в любой электронной схеме, причём их номинальное сопротивление может отличаться не в 2 – 3 раза, а в десятки и сотни раз. Так в схеме можно найти резистор на 1 Ом, и тут же неподалёку на 1000 Ом (1 кОм)!

Поэтому при сборке схемы либо ремонте электронного прибора может потребоваться резистор с определённым номинальным сопротивлением, а под рукой такого нет. В результате быстро найти подходящий резистор с нужным номиналом не всегда удаётся. Это обстоятельство тормозит процесс сборки схемы или ремонта. Выходом из такой ситуации может быть применение составного резистора.

Для того чтобы собрать составной резистор нужно соединить несколько резисторов параллельно или последовательно и тем самым получить нужное нам номинальное сопротивление. На практике это пригождается постоянно.

Знания о правильном соединении резисторов и расчёте их общего сопротивления выручают и ремонтников, восстанавливающих неисправную электронику, и радиолюбителей, занятых сборкой своего электронного устройства.

Последовательное соединение резисторов

  • В жизни последовательное соединение резисторов имеет вид:
  • Последовательно соединённые резисторы серии МЛТ
  • Принципиальная схема последовательного соединения выглядит так:

На схеме видно, что мы заменяем один резистор на несколько, общее сопротивление которых равно тому, который нам необходим.

Подсчитать общее сопротивление при последовательном соединении очень просто. Нужно сложить все номинальные сопротивления резисторов входящих в эту цепь. Взгляните на формулу.

  1. Общее номинальное сопротивление составного резистора обозначено как Rобщ.
  2. Номинальные сопротивления резисторов включённых в цепь обозначаются как R1, R2, R3,…RN.
  3. Применяя последовательное соединение, стоит помнить одно простое правило:

Из всех резисторов, соединённых последовательно главную роль играет тот, у которого самое большое сопротивление. Именно он в значительной степени влияет на общее сопротивление.

Что это значит?

Так, например, если мы соединяем три резистора, номинал которых равен 1, 10 и 100 Ом, то в результате мы получим составной на 111 Ом.

Если убрать резистор на 100 Ом, то общее сопротивление цепочки резко уменьшиться до 11 Ом! А если убрать, к примеру, резистор на 10 Ом, то сопротивление будет уже 101 Ом.

Как видим, резисторы с малыми сопротивлениями в последовательной цепи практически не влияют на общее сопротивление.

Параллельное соединение резисторов

  • Можно соединять резисторы и параллельно:
  • Два резистора МЛТ-2, соединённых параллельно
  • Принципиальная схема параллельного соединения выглядит следующим образом:

Для того чтобы подсчитать общее сопротивление нескольких параллельно соединённых резисторов понадобиться знание формулы. Выглядит она вот так:

Эту формулу можно существенно упростить, если применять только два резистора. В таком случае формула примет вид:

Есть несколько простых правил, позволяющих без предварительного расчёта узнать, каково должно быть сопротивление двух резисторов, чтобы при их параллельном соединении получить то, которое требуется.

Если параллельно соединены два резистора с одинаковым сопротивлением, то общее сопротивление этих резисторов будет ровно в два раза меньше, чем сопротивление каждого из резисторов, входящих в эту цепочку.

Это правило исходит из простой формулы для расчёта общего сопротивления параллельной цепи, состоящей из резисторов одного номинала. Она очень проста. Нужно разделить номинальное сопротивление одного из резисторов на общее их количество:

Здесь R1 – номинальное сопротивление резистора. N – количество резисторов с одинаковым номинальным сопротивлением.

Ознакомившись с приведёнными формулами, вы скажите, что все они справедливы для расчёта ёмкости параллельно и последовательно соединённых конденсаторов. Да, только в отношении конденсаторов всё действует с точностью до «наоборот”. Узнать подробнее о соединении конденсаторов можно здесь.

Проверим справедливость показанных здесь формул на простом эксперименте.

Возьмём два резистора МЛТ-2 на 3 и 47 Ом и соединим их последовательно. Затем измерим общее сопротивление получившейся цепи цифровым мультиметром. Как видим оно равно сумме сопротивлений резисторов, входящих в эту цепочку.

  1. Замер общего сопротивления при последовательном соединении
  2. Теперь соединим наши резисторы параллельно и замерим их общее сопротивление.
  3. Измерение сопротивления при параллельном соединении
  4. Как видим, результирующее сопротивление (2,9 Ом) меньше самого меньшего (3 Ом), входящего в цепочку. Отсюда вытекает ещё одно известное правило, которое можно применять на практике:

При параллельном соединении резисторов общее сопротивление цепи будет меньше наименьшего сопротивления, входящего в эту цепь.

Что ещё нужно учитывать при соединении резисторов?

Во-первых, обязательно учитывается их номинальная мощность. Например, нам нужно подобрать замену резистору на 100 Ом и мощностью 1 Вт. Возьмём два резистора по 50 Ом каждый и соединим их последовательно. На какую мощность рассеяния должны быть рассчитаны эти два резистора?

Поскольку через последовательно соединённые резисторы течёт один и тот же постоянный ток (допустим 0,1 А), а сопротивление каждого из них равно 50 Ом, тогда мощность рассеивания каждого из них должна быть не менее 0,5 Вт. В результате на каждом из них выделится по 0,5 Вт мощности. В сумме это и будет тот самый 1 Вт.

Данный пример достаточно грубоват. Поэтому, если есть сомнения, стоит брать резисторы с запасом по мощности.

Подробнее о мощности рассеивания резистора читайте тут.

Во-вторых, при соединении стоит использовать однотипные резисторы, например, серии МЛТ. Конечно, нет ничего плохого в том, чтобы брать разные. Это лишь рекомендация.

Главная » Радиоэлектроника для начинающих » Текущая страница

Также Вам будет интересно узнать:

Источник: https://go-radio.ru/connection-of-resistors.html

Параллельное соединение резисторов

Господа, в прошлый раз мы с вами говорили про последовательное сопротивление резисторов. Сегодня я бы хотел вам рассказать про другой возможный вид соединения – параллельное.

Чем различается последовательное и параллельное соединение я уже писал в предыдущей статье.  Но все-таки вытащу сюда картинку из той прошлой статьи, я ж знаю, что вам будет лень ходить по ссылкам .

  • А) – Последовательное соединение
  • В) – Параллельное соединение
  • Рисунок 1 – Последовательное и параллельное соединение
  • Как мы видим из рисунка 1, параллельное соединение – это такое соединение, при котором одни концы всех резисторов соединены в один узел, а другие концы – в другой узел.

Сейчас наша задача будет разобраться, как ведут себя токи, напряжения, сопротивления и мощности при таком подключении. Для этого прошу вас взглянуть на рисунок 2, где подробно разрисован расклад дел для параллельного соединения. Будем полагать, что мы знаем величины R1, R2 и R3, а также величину приложенного к схеме напряжения U. Про токи же мы ничего не знаем.

Рисунок 2 – Параллельное соединения

Что мы видим на рисунке 2? Ну, в первую очередь – два узла А и B. В узел А сходятся одни концы всех резисторов, а в узел В – другие концы. Пусть узел А имеет потенциал φ1, а узел В – потенциал φ2. Из рисунка 2 видно, что для всех резисторов R1, R2 и R3 у нас одна и та же разность потенциалов U.

Как следует из статьи про потенциалы, это означает, что напряжение на всех резисторах у нас одинаково и равно приложенному напряжению U. Это важный вывод, его следует хорошо запомнить.

С токами дело обстоит по-другому. Проанализируем рисунок 2 слева направо. Пусть у нас в цепи течет ток I. Течет он себе, течет, никого не трогает и тут вдруг натыкается на узел А. Что в этом случае говорит полюбившаяся вам статья про первый закон Кирхгофа? А то, что ток I в узле А разделится на три тока I1, I2, I3. При этом будет выполняться равенство

То есть через резистор R1 будет протекать ток I1, через резистор R2 – ток I2, а через резистор R3 – ток I3.

Итак, у нас в системе уже тихо-мирно текут себе три тока. И все хорошо, пока они не наткнуться на узел В. Тут снова вступает в силу первый закон Кирхгофа. Эти три тока I1, I2, I3 вновь соединятся в один ток I. Причем после узла В ток будет иметь такую же величину I, какой он был до узла А.

То есть если все вышесказанное воплотить в лаконичный язык наскальной живописи, положение дел можно представить себе вот так

Как же найти эти самые токи I1, I2, I3? Господа, полагаю, вы уже догадались, что на помощь нам придет горячо нами всеми любимый закон Ома. Действительно, мы знаем сопротивления резисторов и, кроме того, нам известно, что на всех них падает одно и тоже напряжение U. Поэтому легко находим токи

Отлично, мы разобрались с напряжениями и с токами в такой схеме. А помните в статье про последовательное сопротивление мы ловко преобразовали три резистора в один с эквивалентным им сопротивлением? Нельзя ли и здесь сделать что-то подобное? Оказывается, вполне себе можно. Как мы помним, токи в схеме распределены таким вот образом

  1. Обзовем эквивалентное сопротивление буковкой R. И подставим в это выражение только что найденные нами токи I1, I2, I3
  2. Видим, что здесь без проблем можно сократить левую и правую части на U. Получаем
  3. Господа, важный вывод: при параллельном соединении резисторов обратное эквивалентное сопротивление равно сумме обратных сопротивлений отдельных резисторов.
  4. То есть для упрощения различных расчетов электрических схем такую вот цепочку параллельно соединенных резисторов можно заменить одним резистором с соответствующим сопротивлением, как показано на рисунке 3.
  5. Рисунок 3 – Преобразование параллельного соединение

Весьма частый случай на практике, когда соединены параллельно не много резисторов, а всего два. Поэтому полезно знать наизусть итоговое сопротивление такой схемы. Давайте посмотрим, чему оно равно:

То есть, если у вас два сопротивления соединены параллельно, то по этой формуле вы легко высчитаете общее сопротивление. Рассмотрим пример. Пусть у нас параллельно соединены два резистора 10 кОм и 15 кОм. Чему равно их общее сопротивление?

Заметьте, господа, итоговое сопротивление у нас получилось 6 кОм, что меньше 10 кОм и 15 кОм. То есть при параллельном соединении общее сопротивление меньше любого из составляющих.

Это всегда верно для любого количества резисторов, а не только для двух. Итоговое сопротивление всегда уменьшается (в отличии от последовательного сопротивления, где итоговое сопротивление всегда растет).

Этот факт полезно запомнить.

Еще один часто встречающийся на практике случай – когда параллельно соединены несколько резисторов с одинаковым сопротивлением. Допустим, каждый из них обладает сопротивлением R1 и всего их N штук. Тогда по нашей общей формуле для эквивалентного сопротивления

  • То есть при параллельном соединении N одинаковых резисторов с сопротивлением R1 итоговое сопротивление будет в N раз меньше этого самого сопротивления R1.
  • Так-с, с током разобрались, с напряжением разобрались, с эквивалентным сопротивлением вроде тоже…осталась мощность. Для этого воспользуемся вот этим выражением, которое мы писали чуть выше в статье
  • Умножим левую и правую части на напряжение U.
  • Как мы помним из статьи про мощность произведение тока на напряжение есть мощность. То есть мы можем записать
  • где Р – мощность, выдаваемая источником;
  • P1 – мощность, рассеиваемая на резисторе R1;
  • P2 – мощность, рассеиваемая на резисторе R2;
  • P3 – мощность, рассеиваемая на резисторе R3.

Заметьте, господа, формула в точности такая же, как и для случая последовательного соединения резисторов. И там и там мощность, выдаваемая источником, равна сумме мощностей, рассеиваемых на резисторах цепи.

Итак, господа, мы рассмотрели основные соотношения при параллельном соединении резисторов. Теперь осталось поговорить, где это параллельное соединение можно использовать и для чего.

1) Ну, во-первых, параллельное соединение применяют во всех случаях, когда хотят запитать несколько нагрузок от одного источника напряжения. При этом пользуются тем свойством, что при параллельном соединении напряжения на всех нагрузках одинаково.

То есть, допустим, вы берете источник напряжения, выставляете на нем напряжение 5 В и цепляете к этому источнику сразу несколько своих устройств. Узлами А и В в этом случае будут клеммы источника. На каждое из устройств в этом случае придет напряжение 5 В.

Да и все устройства в вашей квартире (лампочки, компьютеры, телевизоры и все прочее) соединены между собой параллельно.

2) Второе возможное применение встречается не так часто, но, думаю, о нем тоже следует рассказать. Допустим, вы делаете какую-то схему, где необходим очень точный подгон сопротивления. Скажем, надо получить сопротивление 6 кОм. Такое сопротивление найти нелегко, их просто не продают. Зато у вас есть два сопротивления 10 кОм и 15 кОм.

Вы их соединяете параллельно и получаете требуемые 6 кОм. Как показывает практика, 3 параллельных резисторов достаточно для получения итогового результирующего сопротивления требуемого номинала с весьма хорошей точностью. Конечно, таких вещей лучше избегать и, если есть возможность, всегда стараться применять стандартные сопротивления.

Но бывают случаи, когда это невозможно, и тогда приходит на помощь этот метод.

3) Третий пункт будет немного похож на первый. Его суть заключается в следующим. Допустим, нам надо снять с источника питания 10 Вт мощности. А у нас в наличии только резисторы, которые позволяют рассеивать на себе 1 Вт. Что делать? Можно соединить 10 резисторов параллельно и с каждого снимать по 1 Вт. Мы же помним нашу формулу

Конечно, лучше брать не 10 резисторов, а хотя бы 15 и рассеивать на них меньше, чем 1 Вт. Работать на пределе никогда не следует.

Кстати, тут очень вовремя к моменту написания статьи пришли платы с производства! Господа, прошу вас взглянуть на рисунок 4.

Рисунок 4 – Плата нагревателя

На нем изображена плата нагревателя (флешка для масштаба). В чем суть? Имеется весьма сложное устройство, предназначенное для работы в арктических условиях.

Найти же компоненты, которые надежно функционировать при температурах минус 55 градусов и при этом стоят адекватных денег и обладают адекватными размерами бывает непросто. Обычно элементная база в лучшем случае рассчитана на минус 40 градусов.

И было принято решение разработать вот такой вот нагреватель для прогрева чувствительных к холоду аналоговых узлов устройства. Он управляется с микроконтроллера и автоматически включается при температурах меньше минус 40 градусов.

Как вы можете видеть из рисунка 4, этот нагреватель представляет собой 30 параллельно соединенных резисторов с сопротивлениями 150 Ом. Каждый резистор, согласно документации, способен рассеивать до 1 Вт мощности. Используя изученные формулки, мы можем посчитать, что в сумме такая система обладает сопротивлением

  1. и теоретически может рассеивать мощность

Ну, с сопротивлением вопросов нет, оно действительно равно 5 Ом. Ну, плюс-минус 5 % на допуск резисторов, что в данном случае вообще не критично. А вот с мощностью тут не так все однозначно. Помните про закон Джоуля-Ленца, который мы рассматривали? Резисторы будут греться, причем не слабо.

Как показывает практика, если нагружать резисторы по полной, то есть рассеивать на каждом по 1 Вт, то в течении нескольких секунд их температура улетит за 150 градусов. Такая высокая температура критична для резистора и может привести к его разрушению.

Я был готов к такому развитию событий, поэтому заложил для платы нагревателя максимальное напряжение 9 вольт. Это значит, что на каждом резисторе будет выделяться

  • что почти в два раза меньше максимально допустимой мощности в 1 Вт. В сумме на всей плате выделялось, соответственно

Эксперимент показал, что резисторы достигли температуры с комнатных 25 градусов до критичных 120 градусов приблизительно за 10 секунд работы и температура продолжала уверенно расти.

Очевидно, если оставить на длительное время включенным такой нагреватель при комнатной температуре, он неминуемо выйдет из строя.

Возможно, при работе на минус 55 градусах перегрев бы не был столь критичным, однако хотелось исключить вариант спалить плату на столе, поэтому я понизил напряжение, подаваемое на плату на 3 вольта: стал подавать 6 вольт. Теперь на каждом резисторе рассеивалось

  1. а на всей плате

Теперь температура поднималась до 100-110 градусов примерно за 30-40 секунд работы и оставалась на этом уровне (выходила в точку термодинамического равновесия). Эта температура вполне подходит для нагревателя.

Однако пока это были лишь эксперименты на столе при комнатной температуре, главный эксперимент – в термокамере на минус 55 градусах – впереди. Возможно, по его результатам потребуется чуть увеличить рассеиваемую мощность.

А может все останется как есть и этой мощности будет достаточно для вывода девайса на режим за адекватное время, время покажет .

На сегодня все, господа. Удачи вам и до новых встреч!

Источник: http://myelectronix.ru/postoyannyy-tok/40-parallelnoe-soedinenie-rezistorov

Как отличается параллельное и последовательное соединение резисторов?

Большое разнообразие схем основано на двух видах соединений – последовательное параллельное. Для каждого типа существуют свои собственные законы и принципы. Именно это и позволяет создавать устройства с самыми различными техническими параметрами, в том числе и резисторы. Что же такое резистор?

Резистор – радиодеталь, созданная для контроля напряжения и тока в цепи, увеличивая либо понижая его. Резисторы могут быть двух видов – постоянные и переменные. Так, например, светодиоды требуют для себя совсем небольшого тока. Для этого в электрическую цепочку перед светодиодом устанавливается резистор, который обеспечивает необходимое напряжение для работы последнего.

В статье подробны рассмотрены все аспекты последовательного и параллельного подключения резисторов. Бонусом к статье являются видеоролик и детальная информационная статья на рассматриваемую тему.

Последовательное подключение

Начнем с последовательного соединения. По этой схеме каждый резистор подключается с другим только в одной точке, их может быть в цепи 2, 3 и больше. Обозначим сопротивления: R1, R2, R3 и напряжение источника в цепи Uц. При подключении источника питания в ней начнет протекать ток Iц. В цепи с последовательным соединением ток протекает по всем резисторам один за другим.

Поскольку ток течет через все резисторы их сопротивления и ток суммируется, Iц = I1+I2+I3, Rц = R1 +R2 + R3, чем больше отдельно взятое сопротивление, тем тяжелее электронам преодолевать участок цепи. Мощность резисторов при последовательном и параллельном соединении рассчитывается по разным формулам. В последовательных цепях — складываем, в параллельных — это обратно пропорциональная величина.

Последовательное соединение характеризуется тем, что элементы идут друг за другом. Конец одного подключается к началу другого. При подключении полученной цепочки к источнику тока получается кольцо.

Теоретическая часть

Последовательное соединение характерно тем, что через все элементы протекает ток одинаковой силы. То есть, если цепочка состоит из двух резисторов R1 и R2 (как на рисунке ниже), то ток протекающий через каждое из них и любую другую часть цепи будет одинаковой (I = I1 = I2).

Суммарное сопротивление всей цепи последовательно соединенных резисторов считается как сумма сопротивлений всех ее элементов. То есть, номиналы складывают. R = R1 + R2 — это и есть формула расчета сопротивления при последовательном соединении резисторов. Если элементов больше двух, будет просто больше слагаемых.

Еще одно свойство последовательного соединения — на каждом элементе напряжение отличается. Ток в цепи одинаковый, а напряжение на резисторе зависит от его номинала.

Последовательное подключение.

Примеры расчета

Давайте рассмотрим пример. Цепь представлена на рисунке выше. Есть источник тока и два сопротивления. Пусть R1=1,2 кОм, R2= 800 Ом, а ток в цепи 2 А. По закону Ома U = I * R. Подставляем наши значения:

  • U1 = R1 * I = 1200 Ом * 2 А = 2400 В;
  • U2 = R2 * I = 800 Ом * 2А = 1600 В.

Будет интересно➡  SMD резисторы: что это такое и для чего используются?

Общее напряжение цепи считается как сумма напряжений на резисторах: U = U1 + U2 = 2400 В + 1600 В = 4000 В. Полученную цифру можно проверить. Для этого найдем суммарное сопротивление цепи и умножим его на ток.   R = R1 + R2 = 1200 Ом + 800 Ом = 2000 Ом.

Если подставить в формулу напряжения при последовательном соединении сопротивлений, получаем: U = R * I = 2000 Ом * 2  А = 4000 В. Получаем, что общее напряжение данной цепи 4000 В.

А теперь посмотрите на схему. На первом вольтметре (возле резистора R1) показания будут 2400 В, на втором  — 1600 В.  При этом напряжение источника питания — 4000 В. Последовательное соединение – это соединение двух или более резисторов в форме цепи, в которой каждый отдельный резистор соединяется с другим отдельным резистором только в одной точке.

Материал по теме: Как проверить варистор мультиметром.

Общее сопротивление Rобщ

При таком соединении, через все резисторы проходит один и тот же электрический ток. Чем больше элементов на данном участке электрической цепи, тем «труднее» току протекать через него. Следовательно, при последовательном соединении резисторов их общее сопротивление увеличивается, и оно равно сумме всех сопротивлений.

Параллельное соединение резисторов

Параллельное соединение – это соединение, при котором резисторы соединяются между собой обоими контактами. В результате к одной точке (электрическому узлу) может быть присоединено несколько резисторов.

Параллельное подключение резисторов.

Общее сопротивление Rобщ

При таком соединении, через каждый резистор потечет отдельный ток. Сила данного тока будет обратно пропорциональна сопротивлению резистора. В результате общая проводимость такого участка электрической цепи увеличивается, а общее сопротивление в свою очередь уменьшается.

Таким образом, при параллельном подсоединении резисторов с разным сопротивлением, общее сопротивление будет всегда меньше значения самого маленького отдельного резистора.

Если посмотреть на изображение параллельного соединения, заметно, что ко всем элементам прилагается одинаковое напряжение.

То есть, при параллельном соединении резисторов, на каждом из них будет одинаковое напряжение U = U1 = U2 = U3. Получается, что ток разделяется на несколько «ручейков».

То есть, при параллельном соединении резисторов сила тока, протекающего через каждый из элементов, отличается. I = I1+I2+I3. И зависит сила тока (согласно тому же закону Ома) от сопротивления каждого участка цепи.

В случае с параллельным соединением резисторов — от их номинала.

Предлагаем также почитать интересный материал про малоизвестные факты о двигателях постоянного тока в другой нашей статье.

Схема параллельного соединения

Общее сопротивление участка цепи при таком соединении становится ниже. Его высчитывают по формуле: 1/R = 1/R1 + 1/R + 1/R3+. Такая форма хоть и понятна, но неудобна.

Формула расчета сопротивления параллельно подключенных резисторов получается тем сложнее, чем больше элементов соединены параллельно.

Но больше двух-трех редко кто объединяет, так что на практике достаточно знать только две формулы приведенные ниже.

Если подставить значения в эти формулы, то заметим, что результат будет меньше, чем сопротивление резистора с наименьшим номиналом. Это стоит запомнить: результирующее сопротивление включенных параллельно резисторов будет ниже самого маленького номинала. Давайте сначала рассчитаем параллельное соединение двух резисторов разного номинала и посмотрим что получится.

Соединили параллельно 150 Ом и 100 Ом. Считаем результирующее: 150*100 / (150+100) = 15000/250 = 60 Ом. Если соединить 150 Ом и 50 Ом, получим: 150*50 / (150+50) = 7500 / 200 = 37,5 Ом.

 Как видим, в обоих случаях результат оказывается меньше чем самый низкий номинал соединенных деталей. Этим и пользуются, если в наличии нет сопротивления небольшого номинала.

Проблема только в том, что подбирать сложновато: надо каждый раз считать используя калькулятор.

Как высчитывать сопротивление составных резисторов

Возможно, вам будет проще, если знать, что соединив два одинаковых резистора параллельно, получим результат в два раза меньше. Например, соединив параллельно два резистора по 100 Ом получим составное сопротивление 50 Ом. Проверим? Считаем: 100*100 / (100+100) = 10000 / 200 = 50 Ом. При соединении параллельно трех резисторов, считать приходится больше, так как формула сложнее.

Если подключить параллельно 150 Ом, 100 Ом и 50 Ом, результирующее будет 27,3 Ом. Попробуем с более низкими номиналами. Если параллельно включены 20 Ом, 15 Ом и 10 Ом. Получим результирующее сопротивление 4,61 Ом. Вот вам подтверждение правила. Суммарное сопротивление параллельно соединенных резисторов меньше чем самый низкий номинал.

Параллельное соединение резисторов

Параллельное соединение резисторов это соединение, в котором начала всех резисторов соединены в одну общую точку (А), а концы в другую общую точку. При этом по каждому резистору течет свой ток. При параллельном соединении при протекании тока из точки А в точку Б, он имеет несколько путей.

 Таким образом, увеличение числа параллельно соединенных резисторов ведет к увеличению путей протекания тока, то есть к уменьшению противодействия протеканию тока. А это значит, чем большее количество резисторов соединить параллельно, тем меньше станет значение общего сопротивления такого участка цепи.

Общее сопротивление параллельно соединенных резисторов определяется следующим отношением: 1/Rобщ= 1/R1+1/R2+1/R3+…+1/Rn.

Формулы расчета параллельного и последовательного подключения.

Следует отметить, что здесь действует правило «меньше – меньшего». Это означает, что общее сопротивление всегда будет меньше сопротивления любого параллельно включенного резистора. Общее сопротивление для двух параллельно соединенных резисторов рассчитывается по следующей формуле: Rобщ= R1*R2/R1+R2.

Если имеет место два параллельно соединенных резистора с одинаковыми сопротивлениями, то их общее сопротивление будет равно половине сопротивления одного из них. Параллельное соединение резисторов. При параллельном соединении резисторов нескольких приемников они включаются между двумя точками электрической цепи, образуя параллельные ветви.

Заменяя лампы резисторами с сопротивлениями R1, R2, R3, При параллельном соединении ко всем резисторам приложено одинаковое напряжение U. Поэтому согласно закону Ома: I1=U/R1; I2=U/R2; I3=U/R3.

Источник: https://ElectroInfo.net/radiodetali/rezistory/kak-otlichaetsja-parallelnoe-i-posledovatelnoe-soedinenie-rezistorov.html

Параллельное соединение резисторов: расчет и формулы

В случае последовательного соединения прохождение тока осуществляется только через один проводник. Параллельное соединение резисторов предполагает распределение электрического тока среди нескольких проводников. При добавлении еще одного резистора в электрическую цепь, ток будет частично проходить через разные резисторы.

Схемы последовательного и параллельного соединения

Если рассматривать соединение на примере громкоговорителя, то при последовательном соединении с усилителем мощности подключается только один динамик, поскольку прохождение тока осуществляется только через один проводник. Подключение второго громкоговорителя может быть выполнено разными способами.

При последовательном соединении по обоим устройствам будет протекать одинаковый ток. В этом случае общее сопротивление приборов представляет собой сумму отдельно взятых сопротивлений.

При параллельном соединении протекание тока будет происходить по двум направлениям. Здесь общее значение сопротивления в отличие от последовательного соединения, наоборот, будет уменьшаться. То есть, при параллельном соединении двух сопротивлений, их общее значение будет составлять половину каждого из них.

Если последовательное и параллельное соединение резисторов рассматривается с точки зрения радиоэлектроники, необходимо четко представлять себе, что представляет собой данный элемент и какова его роль в электронных схемах. Эта деталь является неотъемлемой частью многих устройств, благодаря такому свойству, как сопротивление электрическому току. Резисторы могут быть двух типов – постоянными и переменными, то есть подстроечными. При создании тех или иных электрических схем требуется резистор установленного номинала, которого в данный момент может не оказаться в наличии. Поэтому приходится использовать элементы с другими номинальными значениями, формула для каждого из которых подтверждает их физические свойства.

Последовательное соединение считается наиболее простым. Оно используется, когда необходимо увеличить общее сопротивление электрической цепи. В этом случае все сопротивления резисторов просто складываются и дают общую сумму. При параллельном соединении, наоборот, можно снизить результирующее сопротивление или увеличить мощность за счет нескольких подключенных резисторов.

Отличие параллельного и последовательного соединения

Последовательное и параллельное соединение резисторов отличаются между собой значениями напряжения. В каждой части параллельных контуров этот показатель будет одинаковым. Однако, при одном и том же напряжении, сила тока в контурах будет разной. Кроме того, сопротивление резисторов при параллельном соединении будет существенно отличаться от того же показателя при последовательном соединении.

В процессе использования последовательной схемы наблюдаются обратные явления. Сила тока в каждом сопротивлении будет одна и та же, а напряжение на каждом участке будет отличаться. Это связано с тем, что во время протекания тока, каждый резистор частично забирает приложенное напряжение. Из-за различного сопротивления резисторов, при последовательном соединении, напряжение в цепи может падать. Для того чтобы подтвердить данное явление, выполняется расчет сопротивления. Все падения напряжения в общей сумме равняются общему напряжению, которое было приложено. Для проведения вычислений используются формулы, с помощью которых можно получить наиболее точные результаты.

Таким образом, параллельное соединение резисторов, находящихся под одинаковым напряжением, не влияет на режим работы каждого из них. То есть, они совершенно не зависят друг от друга, и ток, проходящий по одному приемнику, не может существенно влиять на другие приемники.

Формула расчета параллельного соединения резисторов

Свои особенности имеет и ток при параллельном соединении резисторов. Попадая в первый узел соединения, он разделяется на столько частей, сколько имеется резисторов, подключенных параллельно. То есть, через сопротивление R1 будет протекать ток I1, а через R2 – ток I2. При попадании во второй узел, они вновь соединяются в один общий ток: I = I1 + I2.

Если какой-либо резистор вышел из строя, то остальные будут нормально функционировать. В этом заключается основное преимущество параллельного соединения. Особенно, это касается двигателей и электрических ламп, работающих от определенного номинального напряжения.

Расчет общего номинального сопротивления осуществляется с помощью формулы: R(общ)=1/(1/R1+1/R2+1/R3+1/R n), где R(общ) – является общим сопротивлением, а R1, R2, R3 и Rn – параллельно подключенными резисторами. Если выполняется параллельное соединение двух резисторов, при котором используется всего лишь два элемента, то в этом случае для расчетов используется следующая схема: R(общ)=R1хR2/R1+R2.

Очень часто в радиоэлектронике приходится пользоваться следующим правилом: если резисторы, подключенные параллельно, имеют один и тот же номинал, то итоговое сопротивление высчитывается путем деления номинала на число подключенных элементов. Такое параллельное соединение резисторов формула представляется следующим образом: R(общ)=R1\n, где R(общ) представляет собой сопротивление, R – номинал параллельно подключенного резистора, n – число подключенных элементов.

Для того чтобы рассчитать параллельное соединение резисторов, следует учитывать, что итоговое сопротивление всех подключенных элементов будет всегда ниже, чем сопротивление резистора с самым низким номиналом. В качестве примера можно рассмотреть схему с тремя резисторами, сопротивления которых составляют 30, 100 и 150 Ом. При использовании основной формулы будет получен следующий результат: R(общ)=1/(1/30+1/100+1/150) =1/(0,03+0,01+0,007)=1/0,047=21,28Ом. Таким образом, три резистора, соединенные параллельно, с минимальным номиналом 30 Ом, в итоге дадут общее сопротивление электрической цепи 21,28 Ом.

Онлайн калькулятор

В случае больших объемов вычислений, расчет параллельного соединения резисторов выполняется с помощью онлайн-калькулятора.

Сопротивление при параллельном соединении, формула для расчета сопротивления при параллельном соединении

В этой статье мы разберем, как посчитать общее сопротивление при параллельном соединении сопротивлений. Параллельным соединением сопротивлений называется соединение (рисунок ниже), при котором один зажим каждого из сопротивлений присоединяется к одной точке (узлу) электрической цепи, а другой зажим каждого из тех же сопротивлений присоединяется к другой точке электрической цепи. Таким образом, между двумя точками (узлами) электрической цепи включается несколько сопротивлений, образующих параллельные ветви.

При этом напряжение между концами всех ветвей будет одним и тем же, а токи в отдельных ветвях определяются по закону Ома:
I1 = U / r1 ; I2 = U / r2 ; I3 = U / r3.

Напряжение U между узлами (А и Б):
U = I1r1 = I2r2 = I3r3,
откуда
I1 / I2 = R2 / R1  и  I2 / I3 = R3 / R2,
т. е.

Токи в параллельных ветвях распределяются обратно пропорционально их сопротивлениям.

Согласно первому правилу Кирхгофа,
I = I1 + I2 + I3
или
U / Rсум = U / R1 + U / R2 + U / R2 = U (1 / R1 + 1 / R2 + 1 / R3).
Произведя сокращение на U, получим:
1 / Rсум = 1 / R1 + 1 / R2 + 1 / R3
или
g = g1 + g2 + g3 ,

где R и g—сопротивление и проводимость разветвленной цепи или, как их часто называют, общие сопротивление и проводимость цепи.
Из полученной формулы следует, что

Общая проводимость разветвленной цепи равна сумме проводимостей отдельных ветвей.

Формула
1 / Rсум = 1 / R1 + 1 / R2 + 1 / R3
дает возможность определить общее сопротивление цепи. Например, для трех параллельно соединенных сопротивлений, приведя правую часть уравнения к общему знаменателю, получим:
1 / Rсум = R2R3 + R1R3 + R1R2 / R1R2R3
откуда
Rсум = R1R2R3 / R2R3 + R1R3 + R1R2
Если сопротивления R1 = R2 = R3, то общее сопротивление цепи:
Rсум = R1 / 3,
а в общем случае при n параллельных ветвях с равными сопротивлениями R1 :
Rсум = R1 / n
В случае двух параллельных ветвей:
1 / Rсум = 1 / R1 + 1 / R2
откуда
Rсум = R1R2 / R2R3 + R1R3

При параллельном соединении приемников энергии все они находятся под одним и тем же напряжением, и режим работы каждого из них не зависит от остальных. Совершенно иначе обстоит дело при последовательном соединении приемников, при котором изменение сопротивления одного из них тотчас же приводит к изменению напряжения на других, последовательно соединенных с ним. Поэтому электрические лампы и двигатели, предназначенные для работы при определенном (номинальном) напряжении, включаются параллельно. Одинаковые электрические лампы иногда соединяются последовательно. Пусть, например, напряжение сети U, а напряжение лампы U0 < U Тогда n ламп соединяются цепочкой друг за другом, причем n > U / U0
Такое соединение ламп можно встретить, например, в трамваях, метро и других случаях.

Пример 1:
К сети с напряжением 220 в параллельно подключены двигатель мощностью 1,1 квт и 11 ламп, каждая мощностью 40 вт. Определить ток в главных (подводящих) проводах

Ток двигателя
I1 = P1 / U = 1100 / 220 = 5a.
Ток ламп
I2 = P2 / U = 11 x 40 / 220 = 2a.
Ток в подводящих проводах
I = I1 + I2 = 5 + 2 = 7a.

Пример 2:
Определить общее сопротивление десяти параллельно включенных ламп накаливания, если каждая из них 240 ом:
R = Rл / n = 240 / 10 = 24ом.

Последовательное и параллельное соединение резисторов


Последовательное соединение резисторов

Последовательное соединение резисторов применяется для увеличения сопротивления. Т.е. когда резисторы соединены последовательно, общее сопротивление равняется сумме сопротивлений каждого резистора. Например, если резисторы R1 и R2 соединены последовательно, их общее сопротивление высчитывается по формуле:
R = R1 + R2.
Это справедливо и для большего количества соединённых последовательно резисторов:
R = R1 + R2 + R3 + R4 + … + Rn.

 

Цепь из последовательно соединённых резисторов будет всегда иметь сопротивление большее, чем у любого резистора из этой цепи.

При последовательном соединении резисторов изменение сопротивления любого резистора из этой цепи влечёт за собой как изменение сопротивления всей цепи так и изменение силы тока в этой цепи.

Параллельное соединение резисторов (формула)

Параллельное соединение резисторов необходимо для уменьшения общего сопротивления и, как вариант, для увеличения мощности нескольких резисторов по сравнению с одним.

Расчет параллельного сопротивления

Расчет параллельного сопротивления двух параллельно соединённых резисторов R1 и R2 производится по следующей формуле:

 

Сопротивление из
двух резисторов:  
R = R1 × R2
 R1 + R2

Параллельное соединение трёх и более резисторов требует более сложной формулы для вычисления общего сопротивления:

Сопротивление параллельных резисторов

 1   =   1 + 1 + 1 + …
RR1R2R3

Как видно, вычислить сопротивление двух параллельных резисторов значительно удобнее.

Сопротивление параллельно соединённых резисторов будет всегда меньше, чем у любого из этих резисторов.

Параллельное соединение резисторов часто используют в случаях, когда необходимо сопротивление с большей мощностью. Для этого, как правило, используют резисторы с одинаковой мощностью и одинаковым сопротивлением. Общая мощность, в таком случае, вычисляется умножением мощности одного резистора на количество параллельно соединённых резисторов.
Например: десять резисторов номиналом 1 КОм и мощностью 1 Вт каждый, соединённые параллельно будут иметь общее сопротивление 100 Ом и мощность 10 Вт.
При последовательном соединении мощность резисторов также складывается. Т.е. в том же примере, но при последовательном соединении, общее сопротивление будет равно 10 КОм и мощность 10 Вт.

Формула расчета сопротивления при параллельном соединении резисторов

Электрическое сопротивление характеризует свойство проводника препятствовать прохождению через него электрического тока. У каждого материала есть свое удельное сопротивление. Это табличная величина, и условно она считается постоянной.

Условно, потому что во многом эта характеристика зависит от внешних условий, например температуры. Сопротивление же какого-либо конкретного элемента (мы будем говорить о резисторах) складывается из многих факторов, например, из геометрических параметров, а когда речь идет о цепи переменного тока, то в расчеты включают также индуктивное и емкостное сопротивление, но об этом мы расскажем позже. Пока же — немного теории.

Закон Ома

В 1826 году немецкий физик Георг Ом на основе своих опытов вывел закон, согласно которому сила тока на участке цепи прямо пропорциональна напряжению, которое к нему приложено, и обратно пропорциональна сопротивлению участка. Из школьного курса мы знаем этот закон:

I=U/R

Позже он был сформулирован и для полной цепи:

I=ε/(R+r)

Где ε — ЭДС источника, R — сопротивление цепи, а r — сопротивление источника.

Мощность прибора

Электрический заряд при своем перемещении совершает работу. Может быть, это незаметно глазу, но вот пощупать результат этой работы можно: электроприборы у нас греются, а иногда нагрев — это цель, а не побочное явление. Не верите — ну, электроплитки, ТЭНы, утюги как раз это свойство и эксплуатируют. Правда, руками это проверять не советую.

Мощностью у нас называют работу, совершенную за единицу времени. Попробуем вычислить мощность электроприбора, включенного в цепь. Поскольку он обладает сопротивлением, обозначим его R, работу — А, мощность — Р, заряд — Q, а время — Δt. Итак, заряд проходит по цепи под действием напряжения U, которое совершает работу по его перемещению на участке цепи за время Δt:

Р=А/Δt , А=UQ

Р=UQ/Δt

Ну а поскольку Q/Δt — не что иное, как сила тока I, получаем:

Р=UI

Свяжем полученное выражение с законом Ома и получим:

Р=I^2*R, P=U^2/R

Последовательное и параллельное соединение

В реальной жизни мы редко имеем дело с одним проводником и одним источником. Достаточно взглянуть в любую принципиальную электрическую схему, например, такую простенькую:

(это схема микроволновки «Электроника»)

можно увидеть, что элементы в схеме соединены по-разному, но мы покажем вам базовые закономерности, которые работают в цепях.

Правила Кирхгофа

Если взять замкнутую электрическую цепь, по которой течет заряд, то можно определенно сказать: он никуда не денется. Сумма всех зарядов, которые текут в одной цепи, всегда одинакова. Это называется законом сохранения заряда, частным случаем общего закона сохранения (как говорится, если в одном месте что-то убудет, в другом непременно прибудет).

Отсюда мы и выводим тот факт, что в каждом узле цепи сумма токов равна нулю. То есть, если ток «приходит» в точку по ветке и «уходит» по двум — значит, первый равен сумме второго и третьего.

На этой картинке мы видим, что I1+I4=I2+I3

Это называется первым правилом Кирхгофа.

Если наша цепь не будет содержать узлов, значит, ток в ней будет величиной постоянной, а элементы, один за другим поставленные в цепь, будут давать падение напряжения. При этом общее напряжение в цепи останется тем же. Отсюда вытекает второе правило Кирхгофа: сумма напряжений на участках цепи будет равна ЭДС источников тока, входящий в эту цепь. Если у нас источник один, то будет верно равенство:

ε=U1+U2+U3+…+Un

Сумма падений напряжения будет, таким образом, нулевой.

В ситуациях, когда мы имеем дело с переменным током, падение будет наблюдаться на участках с конденсаторами и катушками — в цепях переменного тока у них появляется сопротивление (об этом позже).

Теперь, когда мы познакомились с теоретической частью, можем перейти к более приближенному к суровой реальности вопросу, а именно — расчету последовательного и параллельного соединения резисторов.

Примеры расчетов

Рассчитаем параметры цепей с разным типом соединения.

Как мы видим из рисунка, резисторы соединены один за другим, последовательным способом. Значит, ток в этой цепи — величина постоянная, а напряжение, исходя из второго правила Кирхгофа —

U=U1+U2+U3 /напряжение при последовательном соединении/

Поскольку из закона Ома получается U=IR, то

IR=IR1+IR2+IR3,

следовательно, сопротивление всей цепи

R=R1+R2+R3 /сопротивление при последовательном соединении/

а ее потребляемая мощность

Р=I^2*R

На этой картинке мы видим, что резисторы соединены параллельно друг другу.2/R

Исходя из вышеперечисленных закономерностей, вы сможете рассчитывать самые причудливые соединения резисторов, можете попрактиковаться, взяв в библиотеке задачник.

Типы резисторов

Как уже было сказано ранее, элемент, который ставится в цепь для нагрузки, называется резистором. Ставят его для разных целей, главным образом для того, чтобы изменить тот или иной параметр на участке цепи. Например, понизить напряжение или силу тока, чтобы деталь, стоящая за резистором, не сгорела.

Предприятиями выпускается большой ассортимент таких изделий, и их можно по-разному классифицировать. Номинально резистор имеет то сопротивление, которое указано на нем, а по факту оно может зависеть от напряжения в сети (нелинейность), иметь разброс параметра (иногда до 20% доходит). По применяемой технологии резисторы можно разделить на:

  1. проволочные;
  2. композитные;
  3. металлофольговые;
  4. угольные;
  5. интегральные.

Фактическое сопротивление такого элемента может зависеть от температуры окружающей среды и даже от частоты, если мы имеем дело с переменным током. Дело в том, что часть ассортимента резисторов выполнены по проволочной технологии, то есть фактически они представляют собой мини-катушку. При малых частотах (50 Гц) это в расчет не берется, а вот на высоких (мегагерцы) паразитная индуктивность и индуктивное сопротивление может сказаться на работе схемы. Поэтому при выборе резистора для работы с высокочастотными схемами внимательно смотрите. по какой технологии он сделан. Отдайте предпочтение тонкослойным и композиционным изделиям.

Помимо этого, большое распространение получили переменные резисторы, значение сопротивления которых можно регулировать. Делается это чаще всего отверткой. Необходимость в таких изделиях продиктована разбросом параметров у обычных резисторов, а подстроечный вариант позволяет регулировать сопротивление.

Все вышесказанное актуально для цепей постоянного тока и переменного при невысоких частотах, и все это — при нормальных условиях внешней среды. Расчеты цепей при нарушении этих условий нуждаются в дополнительной корректировке: это связано с ограниченностью действия закона Ома. С чем связаны ограничения? Вот несколько примеров:

  1. при сверхнизких температурах многие проводники проявляют такое интересное явление, как сверхпроводимость;
  2. также сопротивление может разниться при нагревании;
  3. неприменим закон Ома для описания электрического тока в газах;
  4. наконец, обычный резистор можно просто пробить высоким напряжением.

Все это прекрасно работает. Не верите — можете поэкспериментировать у себя дома или провести замеры тестером. Например, изучить елочную гирлянду или показания счетчиков при включенных электроприборах (напомню, что в гирлянде лампочки соединены последовательно, а розетки в доме — параллельно). Удачи!

Как я и обещал в статье про переменные резисторы (ссылка), сегодня речь пойдет о возможных способах соединения резисторов, в частности о последовательном соединении и о параллельном.

Последовательное соединение резисторов.

Давайте начнем с рассмотрения цепей, элементы которой соединены последовательно. И хоть мы и будем рассматривать только резисторы в качестве элементов цепи в данной статье, но правила, касающиеся напряжений и токов при разных соединениях будут справедливы и для других элементов. Итак, первая цепь, которую мы будем разбирать выглядит следующим образом:

Здесь у нас классический случай последовательного соединения – два последовательно включенных резистора. Но не будем забегать вперед и рассчитывать общее сопротивление цепи, а для начала рассмотрим все напряжения и токи. Итак, первое правило заключается в том, что протекающие по всем проводникам токи при последовательном соединении равны между собой:

А для определения общего напряжения при последовательном соединении, напряжения на отдельных элементах необходимо просуммировать:

В то же время, по закону Ома для напряжений, сопротивлений и токов в данной цепи справедливы следующие соотношения:

Тогда для вычисления общего напряжения можно будет использовать следующее выражение:

Но для общего напряжение также справедлив закон Ома:

Здесь – это общее сопротивление цепи, которое исходя из двух формул для общего напряжения равно:

Таким образом, при последовательном соединении резисторов общее сопротивление цепи будет равно сумме сопротивлений всех проводников.

Например для следующей цепи:

Общее сопротивление будет равно:

Количество элементов значения не имеет, правило, по которому мы определяем общее сопротивление будем работать в любом случае &#128578; А если при последовательном соединении все сопротивления равны (), то общее сопротивление цепи составит:

в данной формуле равно количеству элементов цепи.

С последовательным соединением резисторов мы разобрались, давайте перейдем к параллельному.

Параллельное соединение резисторов.

При параллельном соединении напряжения на проводниках равны:

А для токов справедливо следующее выражение:

То есть общий ток разветвляется на две составляющие, а его значение равно сумме всех составляющих. По закону Ома:

Подставим эти выражения в формулу общего тока:

А по закону Ома ток:

Приравниваем эти выражения и получаем формулу для общего сопротивления цепи:

Данную формулу можно записать и несколько иначе:

Таким образом, при параллельном соединении проводников величина, обратная общему сопротивлению цепи, равна сумме величин, обратных сопротивлениям параллельно включенных проводников.

Аналогичная ситуация будет наблюдаться и при большем количестве проводников, соединенных параллельно:

Смешанное соединение резисторов.

Помимо параллельного и последовательного соединений резисторов существует еще смешанное соединение. Из названия уже понятно, что при таком соединении в цепи присутствуют резисторы, соединенные как параллельно, так и последовательно. Вот пример такой цепи:

Давайте рассчитаем общее сопротивление цепи. Начнем с резисторов и – они соединены параллельно. Мы можем рассчитать общее сопротивление для этих резисторов и заменить их в схеме одним единственным резистором :

Теперь у нас образовались две группы последовательно соединенных резисторов:

Заменим эти две группы двумя резисторами, сопротивление которых равно:

Как видите, схема стала уже совсем простой ) Заменим группу параллельно соединенных резисторов и одним резистором :

И в итоге у нас на схеме осталось только два резистора соединенных последовательно:

Общее сопротивление цепи получилось равным:

Таким вот образом достаточно большая схема свелась к простейшему последовательному соединению двух резисторов &#128521;

Тут стоит отметить, что некоторые схемы невозможно так просто преобразовать и определить общее сопротивление – для таких схем нужно использовать правила Кирхгофа, о которых мы обязательно поговорим в будущих статьях. А сегодняшняя статья на этом подошла к концу, до скорых встреч на нашем сайте!

Проверим справедливость показанных здесь формул на простом эксперименте.

Возьмём два резистора МЛТ-2 на 3 и 47 Ом и соединим их последовательно. Затем измерим общее сопротивление получившейся цепи цифровым мультиметром. Как видим оно равно сумме сопротивлений резисторов, входящих в эту цепочку.


Замер общего сопротивления при последовательном соединении

Теперь соединим наши резисторы параллельно и замерим их общее сопротивление.


Измерение сопротивления при параллельном соединении

Как видим, результирующее сопротивление (2,9 Ом) меньше самого меньшего (3 Ом), входящего в цепочку. Отсюда вытекает ещё одно известное правило, которое можно применять на практике:

При параллельном соединении резисторов общее сопротивление цепи будет меньше наименьшего сопротивления, входящего в эту цепь.

Что ещё нужно учитывать при соединении резисторов?

Во-первых, обязательно учитывается их номинальная мощность. Например, нам нужно подобрать замену резистору на 100 Ом и мощностью 1 Вт . Возьмём два резистора по 50 Ом каждый и соединим их последовательно. На какую мощность рассеяния должны быть рассчитаны эти два резистора?

Поскольку через последовательно соединённые резисторы течёт один и тот же постоянный ток (допустим 0,1 А ), а сопротивление каждого из них равно 50 Ом , тогда мощность рассеивания каждого из них должна быть не менее 0,5 Вт . В результате на каждом из них выделится по 0,5 Вт мощности. В сумме это и будет тот самый 1 Вт .

Данный пример достаточно грубоват. Поэтому, если есть сомнения, стоит брать резисторы с запасом по мощности.

Подробнее о мощности рассеивания резистора читайте .

Во-вторых, при соединении стоит использовать однотипные резисторы, например, серии МЛТ. Конечно, нет ничего плохого в том, чтобы брать разные. Это лишь рекомендация.

Параллельное соединение резисторов — одно из двух видов электрических соединений, когда оба вывода одного резистора соединены с соответствующими выводами другого резистора или резисторов. Зачастую или параллельно для того, чтобы создать более сложные электронные схемы.

Схема параллельного соединения показан на рисунке ниже. При параллельном соединении резисторов, напряжение на всех резисторах будет одинаковым, а протекающий через них ток будет пропорционален их сопротивлению:

Формула параллельного соединения резисторов

Общее сопротивление нескольких резисторов соединенных параллельно определяется по следующей формуле:

Ток, протекающий через отдельно взятый резистор, согласно , можно найти по формуле:

Параллельное соединение резисторов — расчет

Пример №1

При разработке устройства, возникла необходимость установить резистор с сопротивлением 8 Ом. Если мы просмотрим весь номинальный ряд стандартных значений резисторов, то мы увидим, что резистора с сопротивлением в 8 Ом в нем нет.

Выходом из данной ситуации будет использование двух параллельно соединенных резисторов. Эквивалентное значение сопротивления для двух резисторов соединенных параллельно рассчитывается следующим образом:

Данное уравнение показывает, что если R1 равен R2, то сопротивление R составляет половину сопротивления одного из двух резисторов. При R = 8 Ом, R1 и R2 должны, следовательно, иметь значение 2 × 8 = 16 Ом.
Теперь проведем проверку, рассчитав общее сопротивление двух резисторов:

Таким образом, мы получили необходимое сопротивление 8 Ом, соединив параллельно два резистора по 16 Ом.

Пример расчета №2

Найти общее сопротивление R из трех параллельно соединенных резисторов:

Общее сопротивление R рассчитывается по формуле:

Этот метод расчета может быть использованы для расчета любого количества отдельных сопротивлений соединенных параллельно.

Один важный момент, который необходимо запомнить при расчете параллельно соединенных резисторов – это то, что общее сопротивление всегда будет меньше, чем значение наименьшего сопротивления в этой комбинации.

Как рассчитать сложные схемы соединения резисторов

Более сложные соединения резисторов могут быть рассчитаны путем систематической группировки резисторов. На рисунке ниже необходимо посчитать общее сопротивление цепи, состоящей из трех резисторов:


Для простоты расчета, сначала сгруппируем резисторы по параллельному и последовательному типу соединения.
Резисторы R2 и R3 соединены последовательно (группа 2). Они в свою очередь соединены параллельно с резистором R1 (группа 1).

Последовательное соединение резисторов группы 2 вычисляется как сумма сопротивлений R2 и R3:

В результате мы упрощаем схему в виде двух параллельных резисторов. Теперь общее сопротивление всей схемы можно посчитать следующим образом:

Расчет более сложных соединений резисторов можно выполнить используя законы Кирхгофа.

Ток, протекающий в цепи параллельно соединенных резисторах

Общий ток I протекающий в цепи параллельных резисторов равняется сумме отдельных токов, протекающих во всех параллельных ветвях, причем ток в отдельно взятой ветви не обязательно должен быть равен току в соседних ветвях.

Несмотря на параллельное соединение, к каждому резистору приложено одно и то же напряжение. А поскольку величина сопротивлений в параллельной цепи может быть разной, то и величина протекающего тока через каждый резистор тоже будет отличаться (по определению закона Ома).

Рассмотрим это на примере двух параллельно соединенных резисторов. Ток, который течет через каждый из резисторов (I1 и I2) будет отличаться друг от друга поскольку сопротивления резисторов R1 и R2 не равны.
Однако мы знаем, что ток, который поступает в цепь в точке «А» должен выйти из цепи в точке «B» .

Первое правило Кирхгофа гласит: «Общий ток, выходящий из цепи равен току входящий в цепь».

Таким образом, протекающий общий ток в цепи можно определить как:

Затем с помощью закона Ома можно вычислить ток, который протекает через каждый резистор:

Ток, протекающий в R1 = U ÷ R1 = 12 ÷ 22 кОм = 0,545 мА

Ток, протекающий в R 2 = U ÷ R2 = 12 ÷ 47 кОм = 0,255 мА

Таким образом, общий ток будет равен:

I = 0,545 мА + 0,255 мА = 0,8 мА

Это также можно проверить, используя закон Ома:

I = U ÷ R = 12 В ÷ 15 кОм = 0,8 мА (то же самое)

где 15кОм — это общее сопротивление двух параллельно соединенных резисторов (22 кОм и 47 кОм)

И в завершении хочется отметить, что большинство современных резисторов маркируются цветными полосками и назначение ее можно узнать .

Параллельное соединение резисторов — онлайн калькулятор

Чтобы быстро вычислить общее сопротивление двух и более резисторов, соединенных параллельно, вы можете воспользоваться следующим онлайн калькулятором:

Подведем итог

Когда два или более резистора соединены так, что оба вывода одного резистора соединены с соответствующими выводами другого резистора или резисторов, то говорят, что они соединены между собой параллельно. Напряжение на каждом резисторе внутри параллельной комбинации одинаковое, но токи, протекающие через них, могут отличаться друг от друга, в зависимости от величины сопротивлений каждого резистора.

Эквивалентное или полное сопротивление параллельной комбинации всегда будет меньше минимального сопротивления резистора входящего в параллельное соединение.

На практике нередко встречается задача нахождения сопротивления проводников и резисторов при различных способах соединения. В статье рассмотрено, как рассчитывается сопротивление при и некоторые другие технические вопросы.

Сопротивление проводника

Все проводники имеют свойство препятствовать течению электрического тока, его принято называть электрическим сопротивлением R, оно измеряется в омах. Это основное свойство проводниковых материалов.

Для ведения электротехнических расчётов применяется удельное сопротивление – ρ Ом·м/мм 2 . Все металлы – хорошие проводники, наибольшее применение получили медь и алюминий, гораздо реже применяется железо. Лучший проводник – серебро, оно применяется в электротехнической и электронной промышленности. Широко распространены сплавы с высоким значением сопротивления.

При расчёте сопротивления используется известная из школьного курса физики формула:

R = ρ · l/S, S – площадь сечения; l – длина.

Если взять два проводника, то их сопротивление при параллельном соединении станет меньше из-за увеличения общего сечения.

и нагрев проводника

Для практических расчётов режимов работы проводников применяется понятие плотности тока – δ А/мм 2 , она вычисляется по формуле:

δ = I/S, I – ток, S – сечение.

Ток, проходя по проводнику, нагревает его. Чем больше δ, тем сильнее нагревается проводник. Для проводов и кабелей разработаны нормы допустимой плотности, которые приводятся в Для проводников нагревательных устройств существуют свои нормы плотности тока.

Если плотность δ выше допустимой, может произойти разрушение проводника, например, при перегреве кабеля у него разрушается изоляция.

Правилами регламентируется производить расчёт проводников на нагрев.

Способы соединения проводников

Любой проводник гораздо удобнее изображать на схемах как электрическое сопротивление R, тогда их легко читать и анализировать. Существует всего три способа соединения сопротивлений. Первый способ самый простой – последовательное соединение.

На фото видно, что полное сопротивление равно: R = R 1 + R 2 + R 3 .

Второй способ более сложный – параллельное соединение. Расчёт сопротивления при параллельном соединении выполняется поэтапно. Рассчитывается полная проводимость G = 1/R, а затем полное сопротивление R = 1/G.

Можно поступить и по-другому, прежде рассчитать общее сопротивление при R1 и R2, после этого повторить операцию и найти R.

Третий способ соединения наиболее сложный – смешанное соединение, то есть присутствуют все рассмотренные варианты. Схема приведена на фото.

Для расчёта этой схемы её следует упростить, для этого заменяют резисторы R2 и R3 одним R2,3. Получается несложная схема.

R2,3,4 = R2,3 · R4/(R2,3 + R4).

Схема становится ещё проще, в ней остаются резисторы, имеющие последовательное соединение. В более сложных ситуациях используется этот же метод преобразования.

Виды проводников

В электронной технике, при производстве проводники представляют собою тонкие полоски медной фольги. Ввиду малой длины сопротивление у них незначительно, им во многих случаях можно пренебречь. Для этих проводников сопротивление при параллельном соединении уменьшается вследствие увеличения сечения.

Большой раздел проводников представляют обмоточные провода. Они выпускаются разных диаметров – от 0,02 до 5,6 миллиметра. Для мощных трансформаторов и электродвигателей выпускаются медные шинки прямоугольного сечения. Иногда при ремонте заменяют провод большого диаметра на несколько параллельно соединённых меньшего размера.

Особый раздел проводников представляют провода и кабели, промышленность предоставляет широчайший выбор марок для самых различных нужд. Нередко приходится заменять один кабель на несколько, меньшего сечения. Причины этого бывают самые различные, например, кабель сечением 240 мм 2 очень трудно прокладывать по трассе с крутыми изгибами. Его заменяют на 2×120 мм 2 , и проблема решена.

Расчёт проводов на нагрев

Проводник нагревается протекающим током, если его температура превысит допустимую, наступает разрушение изоляции. ПУЭ предусматривает расчёт проводников на нагрев, исходными данными для него являются сила тока и условия внешней среды, в которой проложен проводник. По этим данным из таблиц в ПУЭ выбирается рекомендуемое проводника или кабеля).

На практике встречаются ситуации, когда нагрузка на действующий кабель сильно возросла. Существует два выхода ‒ заменить кабель на другой, это бывает дорого, или параллельно ему проложить ещё один, чтобы разгрузить основной кабель. В этом случае сопротивление проводника при параллельном соединении уменьшается, следовательно падает выделение тепла.

Чтобы правильно выбрать сечение второго кабеля, пользуются таблицами ПУЭ, важно при этом не ошибиться с определением его рабочего тока. В этой ситуации охлаждение кабелей будет даже лучше, чем у одного. Рекомендуется рассчитать сопротивление при параллельном соединении двух кабелей, чтобы точнее определить их тепловыделение.

Расчёт проводников на потерю напряжения

При расположении потребителя R н на большом расстоянии L от источника энергии U 1 возникает довольно большое на проводах линии. К потребителю R н поступает напряжение U 2 значительно ниже начального U 1 . Практически в качестве нагрузки выступает различное электрооборудование, подключаемое к линии параллельно.

Для решения проблемы производят расчет сопротивления при параллельном соединении всего оборудования, так находится сопротивление нагрузки R н. Далее следует определить сопротивление проводов линии.

Здесь S – сечение провода линии, мм 2 .

Каждый в этой жизни сталкивался с резисторами. Люди с гуманитарными профессиями, как и все, изучали в школе на уроках физики проводники электрического тока и закон Ома.

С резисторами также имеют дело студенты технических университетов и инженеры различных производственных предприятий. Перед всеми этими людьми, так или иначе, вставала задача расчёта электрической цепи при различных видах соединения резисторов. В данной статье речь пойдёт о расчёте физических параметров, характеризующих цепь.

Виды соединений

Резистор – пассивный элемент , присутствующий в каждой электрической цепи. Он предназначен для того, чтобы сопротивляться электрическому току. Существует два вида резисторов:

Зачем же спаивать проводники друг с другом? Например, если для какой-то электрической цепи нужно определённое сопротивление. А среди номинальных показателей нужного нет. В таком случае необходимо подобрать элементы схемы с определёнными значениями сопротивления и соединить их. В зависимости от вида соединения и сопротивлений пассивных элементов мы получим какое-то определённое сопротивление цепи. Оно называется эквивалентным. Его значение зависит от вида спайки проводников. Существует три вида соединения проводников:

Значение эквивалентного сопротивления в цепи считается достаточно легко. Однако, если резисторов в схеме очень много, то лучше воспользоваться специальным калькулятором, который считает это значение. При ведении расчёта вручную, чтобы не допускать ошибок, необходимо проверять, ту ли формулу вы взяли.

Последовательное соединение проводников

В последовательной спайке резисторы идут как бы друг за другом. Значение эквивалентного сопротивления цепи равно сумме сопротивлений всех резисторов. Особенность схем с такой спайкой заключается в том, что значение тока постоянно . Согласно закону Ома, напряжение в цепи равно произведению тока и сопротивления. Так как ток постоянен, то для вычисления напряжения на каждом резисторе, достаточно перемножить значения. После этого необходимо сложить напряжения всех резисторов, и тогда мы получим значение напряжения во всей цепи.

Расчёт очень простой. Так как с ним имеют дело в основном инженеры-разработчики, то для них не составит труда сосчитать всё вручную. Но если резисторов очень много, то проще воспользоваться специальным калькулятором.

Примером последовательного соединения проводников в быту является ёлочная гирлянда.

Параллельное соединение резисторов

При параллельном соединении проводников эквивалентное сопротивление в цепи считается по-другому. Немного сложнее, чем при последовательном.

Его значение в таких цепях равняется произведению сопротивлений всех резисторов, делённому на их сумму. А также есть и другие варианты этой формулы. Параллельное соединение резисторов всегда снижает эквивалентное сопротивление цепи. То есть, его значение всегда будет меньше, чем наибольшее значение какого-то из проводников.

В таких схемах значение напряжения постоянно . То есть значение напряжения во всей цепи равно значениям напряжений каждого из проводников. Оно задаётся источником напряжения.

Сила тока в цепи равна сумме всех токов, протекающих через все проводники. Значение силы тока, протекающего через проводник. равно отношению напряжения источника к сопротивлению этого проводника.

Примеры параллельного соединения проводников:

  1. Освещение.
  2. Розетки в квартире.
  3. Производственное оборудование.

Для расчёта схем с параллельным соединением проводников лучше пользоваться специальным калькулятором. Если в схеме много резисторов, спаянных параллельно, то гораздо быстрее вы посчитаете эквивалентное сопротивление с помощью этого калькулятора.

Смешанное соединение проводников

Этот вид соединения состоит из каскадов резисторов . Например, у нас есть каскад из 10 проводников, соединённых последовательно, и после него идёт каскад из 10 проводников, соединённых параллельно. Эквивалентное сопротивление этой схемы будет равно сумме эквивалентных сопротивлений этих каскадов. То есть, по сути, здесь последовательное соединение двух каскадов проводников.

Многие инженеры занимаются оптимизацией различных схем. Её целью является уменьшение количества элементов в схеме за счёт подбора других, с подходящими значениями сопротивлений. Сложные схемы разбиваются на несколько небольших каскадов, ведь так гораздо проще вести расчёты.

Сейчас, в двадцать первом веке, инженерам стало гораздо проще работать. Ведь несколько десятилетий назад все расчёты производились вручную. А сейчас программисты разработали специальный калькулятор для расчёта эквивалентного сопротивления цепи. В нём запрограммированы формулы, по которым ведутся расчёты.

В этом калькуляторе можно выбрать вид соединения, и потом ввести в специальные поля значения сопротивлений. Через несколько секунд вы уже увидите это значение.

последовательное, параллельное, смешанное соединение. Расчет сопротивления

Резисторы между собой могут быть соединены двумя основными способами: последовательно и параллельно. Смешанное соединение резисторов является их комбинацией.

Сочетания любых соединений резисторов можно привести к одному резистору, расчетом сопротивления которого (R) мы сейчас займемся.

ПАРАЛЛЕЛЬНОЕ СОЕДИНЕНИЕ РЕЗИСТОРОВ

Давайте рассчитаем общее сопротивление такой цепи (рисунок 1). Для этого нам понадобится закон Ома — I=U/R и закон Кирхгофа — I=I1+I2+..In

С учетом этого имеем:

  • I=U/R
  • I1=U/R1
  • I2=U/R2
  • In=U/Rn
  • U/R=U/R1+U/R2+…U/Rn
  • 1/R=1/R1+1/R2+…1/Rn

Последняя формула является основной для расчета сопротивления цепи параллельно соединенных резисторов. Для двух резисторов ее можно записать более удобно: R=(R1*R2)/(R1+R2).

Отсюда следует, что в случае параллельного соединения двух одинаковых по номиналу резисторов (R1=R2) их общее сопротивление будет вдвое меньше любого из них. Это полезно помнить.

ПОСЛЕДОВАТЕЛЬНОЕ СОЕДИНЕНИЕ РЕЗИСТОРОВ

Используя уже упомянутые законы для цепи последовательно соединенных резисторов (рисунок 2) можем записать:

  • U=I*R
  • I=I1=I2=…In
  • U=U1+U2+…Un
  • I*R=I*R1+I*R2+…I*Rn
  • R=R1+R2+…Rn

То есть общее сопротивление резисторов при последовательном соединении равно сумме их сопротивлений.

СМЕШАННОЕ СОЕДИНЕНИЕ РЕЗИСТОРОВ

Такое соединение всегда можно представить как комбинацию последовательного и параллельного соединений (рис.3).

Расчет общего сопротивления цепи при этом производится поэтапно. В приведенном примере рассчитываем:

  1. последовательное сопротивление резисторов Rпосл=R1+R2
  2. параллельное соединение R=(Rпосл*R3)/(Rпосл+R3)

Безусловно, могут встретиться более сложные варианты, но методика расчета их сопротивления та же.

Несколько слов про то, когда возникает необходимость соединять резисторы тем или иным способом:

  1. Отсутствие «под рукой» резистора нужного номинала. При этом следует помнить, что погрешности резисторов будут суммироваться.

    Например, для рисунка 3.a, если фактическая погрешность R1 составляет +10%, а R2 имеет +15%, то для Rпосл она будет +25%.

    Здесь следует обращать внимание на знак, то есть для -10% и +15% в результате получим +5%.

  2. Необходимость получить большую мощность.

    Здесь надо учесть, что при одинаковых номиналах сопротивлений и мощностей соединяемых резисторов, как при последовательном, так и при параллельном их соединении итоговая мощность будет равна сумме мощностей.

    В противном случае следует ее рассчитать, используя закон Ома и формулу для определения рассеиваемой мощности P=I*U.

Про мощность и номиналы резисторов можно почитать здесь.

© 2012-2020 г. Все права защищены.

Представленные на сайте материалы имеют информационный характер и не могут быть использованы в качестве руководящих и нормативных документов


Расчет сопротивления двух параллельно соединенных резисторов. Последовательное и параллельное соединение резисторов. Формула для расчета параллельного соединения сопротивлений

В каждой электрической схеме присутствует резистор, имеющий сопротивление электрическому току. Резисторы бывают двух типов: постоянные и переменные. Во время разработки любой электрической схемы и ремонта электронных изделий часто приходится применять резистор, обладающий необходимым номиналом.

Несмотря на то что для резисторов предусмотрены различные номиналы , может случиться так, что не будет возможности найти необходимый или же вообще ни один элемент не сможет обеспечить требуемый показатель.

Рассчитать производительность и работу

Угол сдвига фаз вычисляется по изображению указателя. Чтобы иметь возможность определять мощности, поглощаемые схемой, предыдущие формулы используются снова. Для определения работы используются следующие формулы.

Дальнейший интересный контент по теме
Резисторы переменного тока представляют собой омические, индуктивные и емкостные резисторы. Для параллельного подключения таких резисторов в цепи переменного тока применяются разные законы, чем для сопротивлений в цепи постоянного тока. Учитывая это, например, катушку: настоящая катушка имеет как индуктивное, так и омическое сопротивление и поэтому может рассматриваться как последовательная связь чисто индуктивного и чисто омического резистора.

Решением этой проблемы может стать применение последовательного и параллельного соединения. Ознакомившись с этой статьей, вы узнаете об особенностях выполнения расчета и подбора различных номиналов сопротивлений.

Часто при изготовлении какого-либо устройства используют резисторы, которые соединяются в соответствии с последовательной схемой. Эффект от применения такого варианта сборки сводится к увеличению общего сопротивления цепи. Для данного варианта соединения элементов создаваемое ими сопротивление рассчитывается как сумма номиналов. Если же сборка деталей выполняется по параллельной схеме, то здесь потребуется рассчитать сопротивление , используя нижеописанные формулы.

Примеры параллельного соединения проводников

Мы рассматриваем только индуктивную составляющую резистора, т.е. катушку как чисто индуктивный резистор. Аналогично, используется омическое сопротивление и емкостное сопротивление, поскольку омическое сопротивление также может иметь индуктивный компонент. В то время как в случае сопротивления проволоки витки, подобные виткам, видны напрямую, это обычно скрыто в резисторах слоя. Фактически, проводящий слой наносят на носитель, из которого материал, проводящий материал, удаляется с помощью процесса спирально-циркулирующего фрезерования, так что остается спирально циркулирующий слой.

К схеме параллельного соединения прибегают в ситуации, когда стоит задача по снижению суммарного сопротивления, а, помимо этого, увеличения мощности для группы элементов, подключенных по параллельной схеме, которое должно быть больше, чем при их отдельном подключении.

Таким образом генерируется требуемое значение сопротивления. Сразу видно, что эта катушечная структура приводит к индуктивному компоненту. Однако это обычно настолько мало, что его можно пренебречь. Общая обработка взаимосвязи любых резисторов переменного тока невозможна и не требуется с помощью математических знаний, доступных в школе.

Объяснение Подключение серии и параллельное соединение

Ниже приведен упрощенный случай параллельной схемы чисто омического, индуктивного и емкостного резисторов. В этой статье мы рассмотрим параллельное соединение и последовательное соединение резисторов. Давайте сначала уточним, что такое последовательное соединение и что такое параллельное соединение, и где разница между последовательным соединением и параллельным соединением. В последовательной цепи мы имеем два или более сопротивления последовательно. Тот же ток протекает через все резисторы.

Расчет сопротивления

В случае подключения деталей друг с другом, с применением параллельной схемы для расчета суммарного сопротивления, будет использоваться следующая формула:

R(общ)=1/(1/R1+1/R2+1/R3+1/Rn).

  • R1- R3 и Rn – резисторы, подсоединенные по параллельной схеме.

Причем, если цепь создается на основе только двух элементов, то для определения суммарного номинального сопротивления следует использовать такую формулу:

Универсальная схема расчета

На следующем графике показаны резисторы последовательно, два резистора, индивидуально нарисованные в начале, и три резистора под электрической цепью. Напротив, существует параллельное соединение резисторов. Что такое параллельная схема? Теперь, в параллельной цепи, линия распадается, и, следовательно, и ток разлагается. В случае параллельного подключения резисторов во многих случаях впервые рассматривается параллельное соединение двух резисторов. Это выглядит следующим образом, включая формулу для расчета.

R(общ)=R1*R2/R1+R2.

  • R(общ) – суммарное сопротивление;
  • R1 и R2 – резисторы, подсоединенные по параллельной схеме.

Универсальная схема расчета

Применительно к радиотехнике следует уделить внимание одному важному правилу: если подключаемые друг к другу элементы по параллельной схеме имеют одинаковый показатель , то для расчета суммарного номинала необходимо общее значение разделить на число подключенных узлов:

Для трех резисторов в параллельной схеме это будет выглядеть на следующем графике, включая формулу для расчета. Разностное соединение и параллельное соединение. В случае последовательной цепи все резисторы подключаются по одной линии за другой. В случае параллельной схемы, с другой стороны, линия расщепляется, резисторы лежат в отдельных линиях. В последовательной цепи тот же ток протекает через все резисторы, а в случае параллельной цепи ток расщепляется. В случае параллельной схемы одно и то же напряжение подается на каждый резистор, но не в последовательной цепи. Другое примечание: смесь последовательной цепи и параллельной схемы называется групповой схемой.

  • R(общ) – суммарное значение сопротивления;
  • R – номинал резистора, подсоединенного по параллельной схеме;
  • n – число подключенных узлов.

Особое внимание следует обратить на то, что конечный показатель сопротивления в случае использования параллельной схемы подключения обязательно будет меньше по сравнению с номиналом любого элемента, подключаемого в цепь.

Примеры Подключение серий и параллельное соединение

В следующих примерах мы увидим, как рассчитать смесь схемы параллельной цепи и серии. В этой области шаг за шагом должен быть рассчитан набор последовательных схем и параллельных схем. На следующем графике показана смесь последовательного соединения и параллельного соединения. Каково общее сопротивление?

Сначала мы суммируем 20 Ом и 30 Ом, так как здесь имеется параллельная схема. Таким образом, схема выглядит следующим образом. Теперь добавим эту схему, добавив резисторы для вычисления общего сопротивления. Вычислите общее сопротивление следующего контура.

Пример расчета

Для большей наглядности можно рассмотреть следующий пример: допустим, у нас есть три резистора, чьи номиналы соответственно равны 100, 150 и 30 Ом. Если воспользоваться первой формулой для определения общего номинала, то получим следующее:

Прежде всего, вы должны увидеть, что есть короткое замыкание на резисторе с 95 Ом. Поэтому ток течет практически полностью по линии ниже, а 95 Ом не учитывается при расчете полного сопротивления. В противном случае у нас есть сочетание последовательной цепи и параллельной схемы.

Комбинированные последовательные и параллельные схемы

Вам нужно знать, как рассчитать резисторы последовательно, параллельно и комбинацию резисторов параллельно и последовательно? Если вы не хотите жарить свою печатную плату, вы это делаете! Эта статья покажет вам, как это сделать за несколько простых шагов. Это просто образный способ говорить, чтобы понятий было легко понять.

Некоторые факты, которые вы должны учитывать
Любой материал, который проводит электрический ток, имеет удельное сопротивление, которое представляет собой сопротивление материала при прохождении электрического тока.
  • Понять понятие сопротивления.
  • Единицей измерения резисторов является Ом.
Параллельное соединение резисторов характеризуется тем, что входные клеммы каждого из резисторов соединены друг с другом.

R(общ)=1/(1/100+1/150+1/30)=1/(0,01+0,007+0,03)=1/0,047=21,28Ом.

Если выполнить несложные расчеты, то можно получить следующее: для цепи, включающей в себя три детали, где наименьший показатель сопротивления составляет 30 Ом, результирующее значение номинала будет равно 21,28 Ом. Этот показатель будет меньше минимального значения номинала в цепи практически на 30%.

Аналогично, в конфигурации параллельного резистора выходные клеммы также соединены друг с другом. Из-за этого все резисторы пропускают одинаковое напряжение, т.е. имеют одинаковое падение напряжения. Это связано с тем, что концы каждого из резисторов соединены с одной и той же точкой в ​​цепи, и поэтому они имеют одинаковое напряжение.

Однако общий ток, протекающий через резисторы параллельно, равен сумме интенсивностей, которые проходят каждый резистор. Дифференциация параллельного соединения из последовательного интерфейса проста. В последовательной конфигурации резистора выходной разъем одного подключается к входному разъему следующего.

Важные нюансы

Обычно для резисторов параллельное соединение применяется тогда, когда стоит задача по созданию сопротивления большей мощности. Для ее решения потребуются резисторы, которые должны иметь равные показатели сопротивления и мощности. При таком варианте определить общую мощность можно следующим образом : мощность одного элемента необходимо перемножить с суммарным числом всех резисторов, из которых состоит цепь, подсоединенных друг с другом в соответствии с параллельной схемой.

Вычисление сопротивлений параллельно: формула

Чтобы вычислить эквивалентное сопротивление нескольких подключенных параллельных резисторов, мы должны применить формулу, указанную выше этих строк. Чтобы избежать ошибок в расчетах, лучше всего разделить формулу на два шага. Сначала мы вычисляем сумму обратного для каждого сопротивления и, когда получаем результат, вычисляем его обратно, чтобы знать эквивалентное сопротивление.

Решенное сопротивление сопротивлениям параллельно

Например, мы вычислим эквивалентное сопротивление конфигурации, аналогичное той, что мы имеем на следующем рисунке. Первый шаг: вычислить сумму обратного каждого сопротивления. Шаг второй: вычислите обратное только что полученное сопротивление.

Вычисление трех резисторов параллельно
Если мы хотим решить предыдущий пример, но используя наш калькулятор из трех резисторов параллельно в сети, просто заполните значение каждого резистора в соответствующем поле. Порядок, в котором вы его пишете, не имеет значения, поэтому вам не нужно его уважать.

Скажем, если нами будут использоваться пять резисторов, чей номинал составляет 100 Ом, а мощность каждого равна 1 Вт, которые присоединены друг к другу в соответствии с параллельной схемой, то суммарный показатель сопротивления будет равен 20 Ом, а мощность составит 5 Вт.

Если взять те же резисторы, но подсоединить их в соответствии с последовательной схемой, то конечная мощность составит 5 Вт, а суммарный номинал будет равен 500 Ом.

Когда вы пишете значение трех резисторов параллельно, просто нажмите кнопку расчета, и вы автоматически получите результат без применения формулы для расчета сопротивления параллельно. С этим вы экономите время и, прежде всего, просчеты. Как мы видели в предыдущих разделах, устройства, которые выступают против прохода электрического тока более выраженным образом, чем обычно, обычно используются в электрических цепях. Эти устройства называются резисторами и могут быть связаны таким образом, что вместе они эквивалентны значению другого сопротивления, называемого эквивалентным сопротивлением.

Заключение

Параллельная схема подключения резисторов очень востребована по той причине, что часто возникает задача по созданию такого номинала, которого невозможно добиться при помощи простого параллельного соединения. При этом процедура расчета этого параметра отличается достаточной сложностью , где необходимо учитывать разные параметры.

Ассоциация резисторов в серии

Он называется полученным сопротивлением или эквивалентом, к значению сопротивления, которое получается путем связывания их набора. В основном резисторы могут быть связаны последовательно, параллельно или комбинацией обоих смешанных вызовов. Когда два или более резисторов последовательно, интенсивность тока, проходящая через каждую из них, одинакова.

Если применить закон Ома к каждому из сопротивлений предыдущего рисунка, мы получим. Если мы сделаем сумму от члена к элементу по трем уравнениям, заметим, что. Таким образом, приведенное выше уравнение, если учесть, что. Итак, если вы понимаете, вы можете видеть, что три предыдущих резистора серии эквивалентны одному резистору, значение которого представляет собой сумму трех предыдущих.

Здесь важная роль отводится не только количеству подключаемых элементов, но и рабочим параметрам резисторов — прежде всего, сопротивлению и мощности. Если один из подключаемых элементов будет иметь неподходящий показатель, то это не позволит эффективно решить задачу по созданию требуемого номинала в цепи.

Последовательное соединение это соединение двух или более резисторов в форме цепи, в которой каждый отдельный резистор соединяется с другим отдельным резистором только в одной точке.

Ассоциация резисторов в параллельном

Когда два или более резисторов параллельны, они делятся своими концами, как показано на следующем рисунке. Если мы применим закон Ома в каждом из сопротивлений фигуры. Зная, что сумма интенсивностей каждого сопротивления равна интенсивности перед входом и выходом из набора, образованного тремя сопротивлениями.

Ассоциация смешанного сопротивления

Как правило, в электрических цепях они не просто похожи на последовательные или параллельные резисторы, но и на комбинацию обоих. Чтобы лучше понять, как подойти к этим типам ассоциаций, мы проиллюстрируем пример. Представьте себе следующую схему сопротивлений.

Общее сопротивление R общ

При таком соединении, через все резисторы проходит один и тот же электрический ток. Чем больше элементов на данном участке электрической цепи, тем «труднее» току протекать через него. Следовательно, при последовательном соединении резисторов их общее сопротивление увеличивается, и оно равно сумме всех сопротивлений.

Подключение 2 равных громкоговорителей последовательно добавляет импедансы и ватты? Сопротивление добавляется, и общая мощность рассеивания на громкоговоритель уменьшается вдвое. Предположим, что выход 8 вольт и динамик 8 Ом ток, который циркулирует громкоговорителем.

Таким образом, динамик должен будет поддерживать более 8 Вт, чтобы он не был поврежден. Теперь подключите два динамика 8 Ом и 8 Вт последовательно, импеданс обоих составляет 16 Ом. Теперь мы вычисляем ток, протекающий через динамики. С этими данными мы вычисляем мощность в каждом динамике.

Напряжение при последовательном соединении

Напряжение при последовательном соединении распределяется на каждый резистор согласно закону Ома:

Т.е чем большее сопротивление резистора, тем большее напряжение на него падает.

Вывод: динамики работают более сдержанно, но усилитель будет поставлять половину мощности, когда динамик составляет 8 Ом. Разделен ли импеданс и ватт параллельно? Если мы рассмотрим одни и те же ораторы на примере предыдущего вопроса, можно сказать, что общий импеданс уменьшается наполовину и рассчитывается следующим образом.

Для работы с более чем двумя динамиками параллельно необходимо использовать другое уравнение для расчета импеданса. Если мы анализируем отдельно каждого динамика, мы понимаем, что каждый из них ведет себя так же, как в примере 1, когда каждый из них подключен к усилителю, через который они будут циркулировать 1 А, а мощность, подлежащая рассеиванию, будет составлять 8 Вт в каждом динамике.

Параллельное соединение это соединение, при котором резисторы соединяются между собой обоими контактами. В результате к одной точке (электрическому узлу) может быть присоединено несколько резисторов.

Общее сопротивление R общ

При таком соединении, через каждый резистор потечет отдельный ток. Сила данного тока будет обратно пропорциональна сопротивлению резистора. В результате общая проводимость такого участка электрической цепи увеличивается, а общее сопротивление в свою очередь уменьшается.

Таким образом, при параллельном подсоединении резисторов с разным сопротивлением, общее сопротивление будет всегда меньше значения самого маленького отдельного резистора.

Формула общей проводимости при параллельном соединении резисторов:

Формула эквивалентного общего сопротивления при параллельном соединении резисторов:

Для двух одинаковых резисторов общее сопротивление будет равно половине одного отдельного резистора:

Соответственно, для n одинаковых резисторов общее сопротивление будет равно значению одного резистора, разделенного на n.

Напряжение при параллельном соединении

Напряжение между точками A и B является как общим напряжением для всего участка цепи, так и напряжением, падающим на каждый резистор в отдельности. Поэтому при параллельном соединении на все резисторы упадет одинаковое напряжение.

Через каждый резистор течет ток, сила которого обратно пропорциональна сопротивлению резистора. Для того чтобы узнать какой ток течет через определенный резистор, можно воспользоваться законом Ома:

Смешанным соединением называют участок цепи, где часть резисторов соединяются между собой последовательно, а часть параллельно. В свою очередь, смешанное соединение бывает последовательного и параллельного типов.

Общее сопротивление R общ

  • Цепь разбивают на участки с только пареллельным или только последовательным соединением.
  • Вычисляют общее сопротивление для каждого отдельного участка.
  • Вычисляют общее сопротивление для всей цепи смешанного соединения.

Также существует более быстрый способ расчета общего сопротивления для смешанного соединения. Можно, в соответствии схеме, сразу записывать формулу следующим образом:

  • Если резисторы соединяются последоватеьно — складывать.
  • Если резисторы соединяются параллельно — использовать условное обозначение «||».
  • Подставлять формулу для параллельного соединения где стоит символ «||».

Так это будет выглядеть для схемы 1:

Как рассчитать последовательные и параллельные резисторы — Kitronik Ltd

Резисторы серии

Когда резисторы подключаются друг за другом, это называется последовательным соединением. Это показано ниже. Чтобы рассчитать общее общее сопротивление ряда резисторов, подключенных таким образом, вы складываете отдельные сопротивления. Это делается по следующей формуле: Rtotal = R1 + R2 + R3 и так далее. Пример: чтобы рассчитать полное сопротивление для этих трех последовательно соединенных резисторов.
Rtotal = R1 + R2 + R3 = 100 + 82 + 1 Ом = 183 Ом

Задача 1:

Рассчитайте общее сопротивление следующего последовательно включенного резистора.
R Итого = _______________
= _______________
R Итого = _______________
= _______________
R Итого = _______________
= _______________

Параллельные резисторы

Когда резисторы подключаются друг к другу (бок о бок), это называется параллельным подключением.Это показано ниже.

Два параллельных резистора

Для расчета общего полного сопротивления a двух резисторов, подключенных таким образом, вы можете использовать следующую формулу:
Пример: чтобы рассчитать полное сопротивление для этих двух резисторов, включенных параллельно.

Задача 2:

Рассчитайте полное сопротивление следующего резистора, включенного параллельно.

Три или более резистора, включенных параллельно

Для расчета общего общего сопротивления ряда из трех или более резисторов, подключенных таким образом, вы можете использовать следующую формулу: Пример: Чтобы вычислить общее сопротивление для этих трех резисторов, подключенных параллельно

Задача 3:

Рассчитайте полное сопротивление следующего резистора, включенного параллельно.

Ответы

Задача 1

1 = 1492 Ом 2 = 2242 Ом 3 = 4847 Ом

Задача 2

1 = 5 Ом 2 = 9,57 Ом 3 = 248,12 Ом

Задача 3

1 = 5,95 Ом 2 = 23,76 Ом Загрузите pdf-версию этой страницы здесь. Подробнее об авторе подробнее »

© Kitronik Ltd — Вы можете распечатать эту страницу и ссылку на нее, но не должны копировать страницу или ее часть без предварительного письменного согласия Kitronik.

10.3: Последовательные и параллельные резисторы

Цели обучения

К концу раздела вы сможете:

  • Определите термин эквивалентное сопротивление
  • Рассчитайте эквивалентное сопротивление резисторов, включенных последовательно
  • Рассчитайте эквивалентное сопротивление резисторов, включенных параллельно

В книге «Ток и сопротивление» мы описали термин «сопротивление» и объяснили основную конструкцию резистора.По сути, резистор ограничивает поток заряда в цепи и представляет собой омическое устройство, где \ (V = IR \). В большинстве схем имеется более одного резистора. Если несколько резисторов соединены вместе и подключены к батарее, ток, подаваемый батареей, зависит от эквивалентного сопротивления цепи.

Эквивалентное сопротивление комбинации резисторов зависит как от их индивидуальных значений, так и от способа их подключения. Самыми простыми комбинациями резисторов являются последовательное и параллельное соединение (Рисунок \ (\ PageIndex {1} \)).В последовательной схеме выходной ток первого резистора течет на вход второго резистора; следовательно, ток в каждом резисторе одинаков. В параллельной схеме все выводы резистора на одной стороне резисторов соединены вместе, а все выводы на другой стороне соединены вместе. В случае параллельной конфигурации каждый резистор имеет одинаковое падение потенциала на нем, и токи через каждый резистор могут быть разными, в зависимости от резистора.Сумма отдельных токов равна току, протекающему по параллельным соединениям.

Рисунок \ (\ PageIndex {1} \): (a) При последовательном соединении резисторов ток одинаков в каждом резисторе. (b) При параллельном соединении резисторов напряжение на каждом резисторе одинаковое.

Резисторы серии

Считается, что резисторы

включены последовательно, когда ток течет через резисторы последовательно. Рассмотрим рисунок \ (\ PageIndex {2} \), на котором показаны три последовательно включенных резистора с приложенным напряжением, равным \ (V_ {ab} \).Поскольку заряды проходят только по одному пути, ток через каждый резистор одинаков. Эквивалентное сопротивление набора резисторов при последовательном соединении равно алгебраической сумме отдельных сопротивлений.

Рисунок \ (\ PageIndex {2} \): (a) Три резистора, подключенные последовательно к источнику напряжения. (b) Исходная схема сокращается до эквивалентного сопротивления и источника напряжения.

На рисунке \ (\ PageIndex {2} \) ток, исходящий от источника напряжения, протекает через каждый резистор, поэтому ток через каждый резистор одинаков.Ток в цепи зависит от напряжения, подаваемого источником напряжения, и сопротивления резисторов. Для каждого резистора происходит падение потенциала, равное потере электрической потенциальной энергии при прохождении тока через каждый резистор. Согласно закону Ома, падение потенциала \ (V \) на резисторе при протекании через него тока рассчитывается по формуле \ (V = IR \), где \ (I \) — ток в амперах (\ (A \)), а \ (R \) — сопротивление в Ом \ ((\ Omega) \).N V_i = 0. \]

Это уравнение часто называют законом петли Кирхгофа, который мы рассмотрим более подробно позже в этой главе. На рисунке \ (\ PageIndex {2} \) сумма падения потенциала каждого резистора и напряжения, подаваемого источником напряжения, должна равняться нулю:

\ [\ begin {align *} V — V_1 — V_2 — V_3 & = 0, \\ [4pt] V & = V_1 + V_2 + V_3, \\ [4pt] & = IR_1 + IR_2 + IR_3, \ end { выровнять *} \]

Решение для \ (I \)

\ [\ begin {align *} I & = \ frac {V} {R_1 + R_2 + R_3} \\ [4pt] & = \ frac {V} {R_ {S}}.\ end {align *} \]

Поскольку ток через каждый компонент одинаков, равенство можно упростить до эквивалентного сопротивления (\ (R_ {S} \)), которое представляет собой просто сумму сопротивлений отдельных резисторов. N R_i.\ label {серия эквивалентных сопротивлений} \]

Одним из результатов подключения компонентов в последовательную цепь является то, что если что-то происходит с одним компонентом, это влияет на все остальные компоненты. Например, если несколько ламп подключены последовательно и одна лампа перегорела, все остальные лампы погаснут.

Пример \ (\ PageIndex {1} \): эквивалентное сопротивление, ток и мощность в последовательной цепи

Батарея с напряжением на клеммах 9 В подключена к цепи, состоящей из четырех последовательно соединенных резисторов \ (20 \, \ Omega \) и одного \ (10 ​​\, \ Omega \) (Рисунок \ (\ PageIndex {3 } \)).Предположим, что батарея имеет незначительное внутреннее сопротивление.

  1. Рассчитайте эквивалентное сопротивление цепи.
  2. Рассчитайте ток через каждый резистор.
  3. Рассчитайте падение потенциала на каждом резисторе.
  4. Определите общую мощность, рассеиваемую резисторами, и мощность, потребляемую батареей.
Рисунок \ (\ PageIndex {3} \): Простая последовательная схема с пятью резисторами.

Стратегия

В последовательной цепи эквивалентное сопротивление представляет собой алгебраическую сумму сопротивлений.2R \), а общая мощность, рассеиваемая резисторами, равна сумме мощности, рассеиваемой каждым резистором. Мощность, подаваемая батареей, можно найти с помощью \ (P = I \ epsilon \).

Раствор

  1. Эквивалентное сопротивление — это алгебраическая сумма сопротивлений (Уравнение \ ref {серия эквивалентных сопротивлений}): \ [\ begin {align *} R_ {S} & = R_1 + R_2 + R_3 + R_4 + R_5 \\ [4pt ] & = 20 \, \ Омега + 20 \, \ Омега + 20 \, \ Омега + 20 \, \ Омега + 10 \, \ Омега = 90 \, \ Омега.2 (10 \, \ Omega) = 0,1 \, W, \ nonumber \] \ [P_ {рассеивается} = 0,2 \, W + 0,2 \, W + 0,2 \, W + 0,2 \, W + 0,1 \, W = 0,9 \, W, \ nonumber \] \ [P_ {источник} = I \ epsilon = (0,1 \, A) (9 \, V) = 0,9 \, W. \ nonumber \]

Значение

Существует несколько причин, по которым мы использовали бы несколько резисторов вместо одного резистора с сопротивлением, равным эквивалентному сопротивлению цепи. Возможно, резистора необходимого размера нет в наличии, или нам нужно отводить выделяемое тепло, или мы хотим минимизировать стоимость резисторов.Каждый резистор может стоить от нескольких центов до нескольких долларов, но при умножении на тысячи единиц экономия затрат может быть значительной.

Упражнение \ (\ PageIndex {1} \)

Некоторые гирлянды миниатюрных праздничных огней закорачиваются при перегорании лампочки. Устройство, вызывающее короткое замыкание, называется шунтом, который позволяет току течь по разомкнутой цепи. «Короткое замыкание» похоже на протягивание куска проволоки через компонент. Луковицы обычно сгруппированы в серию по девять луковиц.Если перегорает слишком много лампочек, в конце концов открываются шунты. Что вызывает это?

Ответ

Эквивалентное сопротивление девяти последовательно соединенных лампочек составляет 9 R . Ток равен \ (I = V / 9 \, R \). Если одна лампочка перегорит, эквивалентное сопротивление составит 8 R , и напряжение не изменится, но ток возрастет \ ((I = V / 8 \, R \). Чем больше лампочек перегорят, ток станет равным. В конце концов, ток становится слишком большим, что приводит к сгоранию шунта.№ Р_и. \]

  • Одинаковый ток протекает последовательно через каждый резистор.
  • Отдельные последовательно включенные резисторы не получают полное напряжение источника, а делят его. Общее падение потенциала на последовательной конфигурации резисторов равно сумме падений потенциала на каждом резисторе.
  • Параллельные резисторы

    На рисунке \ (\ PageIndex {4} \) показаны резисторы, включенные параллельно, подключенные к источнику напряжения. Резисторы включены параллельно, когда один конец всех резисторов соединен непрерывным проводом с незначительным сопротивлением, а другой конец всех резисторов также соединен друг с другом непрерывным проводом с незначительным сопротивлением.Падение потенциала на каждом резисторе одинаковое. Ток через каждый резистор можно найти с помощью закона Ома \ (I = V / R \), где напряжение на каждом резисторе постоянно. Например, автомобильные фары, радио и другие системы подключены параллельно, так что каждая подсистема использует полное напряжение источника и может работать полностью независимо. То же самое и с электропроводкой в ​​вашем доме или любом здании.

    Рисунок \ (\ PageIndex {4} \): Два резистора, подключенных параллельно источнику напряжения.(b) Исходная схема сокращается до эквивалентного сопротивления и источника напряжения.

    Ток, протекающий от источника напряжения на рисунке \ (\ PageIndex {4} \), зависит от напряжения, подаваемого источником напряжения, и эквивалентного сопротивления цепи. В этом случае ток течет от источника напряжения и попадает в переход или узел, где цепь разделяется, протекая через резисторы \ (R_1 \) и \ (R_2 \). По мере прохождения зарядов от батареи часть заряда проходит через резистор \ (R_1 \), а часть — через резистор \ (R_2 \).Сумма токов, текущих в переход, должна быть равна сумме токов, текущих из перехода:

    \ [\ sum I_ {in} = \ sum I_ {out}. {- 1}.{-1}. \ label {10.3} \]

    Это соотношение приводит к эквивалентному сопротивлению \ (R_ {P} \), которое меньше наименьшего из отдельных сопротивлений. Когда резисторы подключены параллельно, от источника течет больше тока, чем протекает для любого из них по отдельности, поэтому общее сопротивление ниже.

    Пример \ (\ PageIndex {2} \): Анализ параллельной цепи

    Три резистора \ (R_1 = 1,00 \, \ Omega \), \ (R_2 = 2,00 \, \ Omega \) и \ (R_3 = 2,00 \, \ Omega \) подключены параллельно.Параллельное соединение подключается к источнику напряжения \ (V = 3,00 \, V \).

    1. Какое эквивалентное сопротивление?
    2. Найдите ток, подаваемый источником в параллельную цепь.
    3. Рассчитайте токи в каждом резисторе и покажите, что в сумме они равны выходному току источника.
    4. Рассчитайте мощность, рассеиваемую каждым резистором.
    5. Найдите выходную мощность источника и покажите, что она равна общей мощности, рассеиваемой резисторами.

    Стратегия

    (a) Общее сопротивление для параллельной комбинации резисторов находится с помощью уравнения \ ref {10.3}. (Обратите внимание, что в этих вычислениях каждый промежуточный ответ отображается с дополнительной цифрой.)

    (b) Ток, подаваемый источником, можно найти из закона Ома, заменив \ (R_ {P} \) на полное сопротивление \ (I = \ frac {V} {R_ {P}} \).

    (c) Отдельные токи легко вычислить по закону Ома \ (\ left (I_i = \ frac {V_i} {R_i} \ right) \), поскольку каждый резистор получает полное напряжение.{-1} = 0,50 \, \ Omega. \ Nonumber \] Общее сопротивление с правильным количеством значащих цифр равно \ (R_ {eq} = 0,50 \, \ Omega \). Как и предполагалось, \ (R_ {P} \) меньше наименьшего индивидуального сопротивления.

  • Полный ток можно найти из закона Ома, заменив полное сопротивление \ (R_ {P} \). Это дает \ [I = \ frac {V} {R_ {P}} = \ frac {3.00 \, V} {0.50 \, \ Omega} = 6.00 \, A. \ nonumber \] Текущий I для каждого устройства намного больше, чем для тех же устройств, подключенных последовательно (см. предыдущий пример).Схема с параллельным соединением имеет меньшее общее сопротивление, чем резисторы, включенные последовательно.
  • Отдельные токи легко вычислить по закону Ома, поскольку каждый резистор получает полное напряжение. Таким образом, \ [I_1 = \ frac {V} {R_1} = \ frac {3.00 \, V} {1.00 \, \ Omega} = 3.00 \, A. \ nonumber \] Аналогично, \ [I_2 = \ frac {V } {R_2} = \ frac {3.00 \, V} {2.00 \, \ Omega} = 1.50 \, A \ nonumber \] и \ [I_3 = \ frac {V} {R_3} = \ frac {3.00 \, V } {2.00 \, \ Omega} = 1.50 \, A. \ nonumber \] Полный ток — это сумма отдельных токов: \ [I_1 + I_2 + I_3 = 6.2} {2.00 \, \ Omega} = 4.50 \, W. \ nonumber \]
  • Общую мощность также можно рассчитать несколькими способами. Выбор \ (P = IV \) и ввод общей текущей доходности \ [P = IV = (6.00 \, A) (3.00 \, V) = 18.00 \, W. \ nonumber \]
  • Значение

    Полная мощность, рассеиваемая резисторами, также составляет 18,00 Вт:

    \ [P_1 + P_2 + P_3 = 9,00 \, W + 4,50 \, W + 4,50 \, W = 18,00 \, W. \ nonumber \]

    Обратите внимание, что общая мощность, рассеиваемая резисторами, равна мощности, подаваемой источником.

    Упражнение \ (\ PageIndex {2A} \)

    Рассмотрим ту же самую разность потенциалов \ ((V = 3,00 \, V) \), приложенную к одним и тем же трем последовательно включенным резисторам. Будет ли эквивалентное сопротивление последовательной цепи больше, меньше или равно трем резисторам, включенным параллельно? Будет ли ток в последовательной цепи выше, ниже или равен току, обеспечиваемому тем же напряжением, приложенным к параллельной цепи? Как мощность, рассеиваемая последовательно подключенными резисторами, будет сравниваться с мощностью, рассеиваемой параллельно резисторами?

    Раствор

    Эквивалент последовательной схемы будет \ (R_ {eq} = 1.00 \, \ Omega + 2.00 \, \ Omega + 2.00 \, \ Omega = 5.00 \, \ Omega \), что выше эквивалентного сопротивления параллельной цепи \ (R_ {eq} = 0.50 \, \ Omega \ ). Эквивалентный резистор любого количества резисторов всегда выше, чем эквивалентное сопротивление тех же резисторов, соединенных параллельно. Ток через последовательную цепь будет \ (I = \ frac {3.00 \, V} {5.00 \, \ Omega} = 0.60 \, A \), что меньше суммы токов, проходящих через каждый резистор в параллельная цепь, \ (I = 6.00 \, А \). Это неудивительно, поскольку эквивалентное сопротивление последовательной цепи выше. Ток при последовательном соединении любого количества резисторов всегда будет ниже, чем ток при параллельном соединении тех же резисторов, поскольку эквивалентное сопротивление последовательной цепи будет выше, чем параллельной цепи. Мощность, рассеиваемая последовательно включенными резисторами, будет равна \ (P = 1,800 \, Вт \), что ниже мощности, рассеиваемой в параллельной цепи \ (P = 18.00 \, Вт \).

    Упражнение \ (\ PageIndex {2B} \)

    Как бы вы использовали реку и два водопада, чтобы смоделировать параллельную конфигурацию двух резисторов? Как разрушается эта аналогия?

    Раствор

    Река, текущая горизонтально с постоянной скоростью, разделяется на две части и течет через два водопада. Молекулы воды аналогичны электронам в параллельных цепях. Количество молекул воды, которые текут в реке и падает, должно быть равно количеству молекул, которые текут над каждым водопадом, точно так же, как сумма тока через каждый резистор должна быть равна току, текущему в параллельном контуре.Молекулы воды в реке обладают энергией благодаря своему движению и высоте. Потенциальная энергия молекул воды в реке постоянна из-за их одинаковой высоты. Это аналогично постоянному изменению напряжения в параллельной цепи. Напряжение — это потенциальная энергия на каждом резисторе.

    При рассмотрении энергии аналогия быстро разрушается. В водопаде потенциальная энергия преобразуется в кинетическую энергию молекул воды. В случае прохождения электронов через резистор падение потенциала преобразуется в тепло и свет, а не в кинетическую энергию электронов.

    Подведем итог основным характеристикам резисторов параллельно:

    1. Эквивалентное сопротивление находится по формуле \ ref {10.3} и меньше любого отдельного сопротивления в комбинации.
    2. Падение потенциала на каждом параллельном резисторе одинаковое.
    3. Параллельные резисторы не получают суммарный ток каждый; они делят это. Ток, поступающий в параллельную комбинацию резисторов, равен сумме токов, протекающих через каждый резистор, включенный параллельно.

    В этой главе мы представили эквивалентное сопротивление резисторов, соединенных последовательно, и резисторов, соединенных параллельно. Как вы помните, в разделе «Емкость» мы ввели эквивалентную емкость конденсаторов, соединенных последовательно и параллельно. Цепи часто содержат как конденсаторы, так и резисторы. Таблица \ (\ PageIndex {1} \) суммирует уравнения, используемые для эквивалентного сопротивления и эквивалентной емкости для последовательных и параллельных соединений.

    Таблица \ (\ PageIndex {1} \): Сводка по эквивалентному сопротивлению и емкости в последовательной и параллельной комбинациях
    Комбинация серий Параллельная комбинация
    Эквивалентная емкость \ [\ frac {1} {C_ {S}} = \ frac {1} {C_1} + \ frac {1} {C_2} + \ frac {1} {C_3} +.N R_i \ nonumber \] \ [\ frac {1} {R_ {P}} = \ frac {1} {R_1} + \ frac {1} {R_2} + \ frac {1} {R_3} +. . . \ nonumber \]

    Сочетания последовательного и параллельного

    Более сложные соединения резисторов часто представляют собой просто комбинации последовательного и параллельного соединения. Такие комбинации обычны, особенно если учесть сопротивление проводов. В этом случае сопротивление провода включено последовательно с другими сопротивлениями, включенными параллельно.

    Комбинации последовательного и параллельного соединения можно свести к одному эквивалентному сопротивлению, используя технику, показанную на рисунке \ (\ PageIndex {5} \).Различные части могут быть идентифицированы как последовательные или параллельные соединения, уменьшенные до их эквивалентных сопротивлений, а затем уменьшенные до тех пор, пока не останется единственное эквивалентное сопротивление. Процесс более трудоемкий, чем трудный. Здесь мы отмечаем эквивалентное сопротивление как \ (R_ {eq} \).

    Рисунок \ (\ PageIndex {5} \): (а) Исходная схема из четырех резисторов. (b) Шаг 1: резисторы \ (R_3 \) и \ (R_4 \) включены последовательно, и эквивалентное сопротивление равно \ (R_ {34} = 10 \, \ Omega \). (c) Шаг 2: сокращенная схема показывает, что резисторы \ (R_2 \) и \ (R_ {34} \) включены параллельно, с эквивалентным сопротивлением \ (R_ {234} = 5 \, \ Omega \).(d) Шаг 3: сокращенная схема показывает, что \ (R_1 \) и \ (R_ {234} \) включены последовательно с эквивалентным сопротивлением \ (R_ {1234} = 12 \, \ Omega \), которое является эквивалентное сопротивление \ (R_ {eq} \). (e) Уменьшенная схема с источником напряжения \ (V = 24 \, V \) с эквивалентным сопротивлением \ (R_ {eq} = 12 \, \ Omega \). Это приводит к току \ (I = 2 \, A \) от источника напряжения.

    Обратите внимание, что резисторы \ (R_3 \) и \ (R_4 \) включены последовательно. Их можно объединить в одно эквивалентное сопротивление.Один из методов отслеживания процесса — включить резисторы в качестве индексов. {- 1} = 5 \, \ Omega.\ nonumber \]

    Этот шаг процесса сокращает схему до двух резисторов, показанных на рисунке \ (\ PageIndex {5d} \). Здесь схема сводится к двум резисторам, которые в данном случае включены последовательно. Эти два резистора можно уменьшить до эквивалентного сопротивления, которое является эквивалентным сопротивлением цепи:

    \ [R_ {eq} = R_ {1234} = R_1 + R_ {234} = 7 \, \ Omega + 5 \ Omega = 12 \, \ Omega. \ nonumber \]

    Основная цель этого анализа схемы достигнута, и теперь схема сведена к одному резистору и одному источнику напряжения.

    Теперь мы можем проанализировать схему. Ток, обеспечиваемый источником напряжения, равен \ (I = \ frac {V} {R_ {eq}} = \ frac {24 \, V} {12 \, \ Omega} = 2 \, A \). Этот ток проходит через резистор \ (R_1 \) и обозначается как \ (I_1 \). Падение потенциала на \ (R_1 \) можно найти с помощью закона Ома:

    \ [V_1 = I_1R_1 = (2 \, A) (7 \, \ Omega) = 14 \, V. \ nonumber \]

    Глядя на рисунок \ (\ PageIndex {5c} \), это оставляет \ (24 \, V — 14 \, V = 10 \, V \), чтобы отбросить его через параллельную комбинацию \ (R_2 \) и \ ( R_ {34} \).Ток через \ (R_2 \) можно найти по закону Ома:

    \ [I_2 = \ frac {V_2} {R_2} = \ frac {10 \, V} {10 \, \ Omega} = 1 \, A. \ nonumber \]

    Резисторы \ (R_3 \) и \ (R_4 \) включены последовательно, поэтому токи \ (I_3 \) и \ (I_4 \) равны

    .

    \ [I_3 = I_4 = I — I_2 = 2 \, A — 1 \, A = 1 \, A. \ nonumber \]

    Используя закон Ома, мы можем найти падение потенциала на двух последних резисторах. Потенциальные капли равны \ (V_3 = I_3R_3 = 6 \, V \) и \ (V_4 = I_4R_4 = 4 \, V \).2 (4 \, \ Omega) = 4 \, W, \\ [4pt] P_ {рассеивается} & = P_1 + P_2 + P_3 + P_4 = 48 \, W. \ end {align *} \]

    Полная энергия постоянна в любом процессе. Следовательно, мощность, подаваемая источником напряжения, составляет

    \ [\ begin {align *} P_s & = IV \\ [4pt] & = (2 \, A) (24 \, V) = 48 \, W \ end {align *} \]

    Анализ мощности, подаваемой в схему, и мощности, рассеиваемой резисторами, является хорошей проверкой достоверности анализа; они должны быть равны.

    Пример \ (\ PageIndex {3} \): объединение последовательных и параллельных цепей

    На рисунке \ (\ PageIndex {6} \) показаны резисторы, подключенные последовательно и параллельно.Мы можем считать \ (R_1 \) сопротивлением проводов, ведущих к \ (R_2 \) и \ (R_3 \).

    1. Найдите эквивалентное сопротивление цепи.
    2. Какое падение потенциала \ (V_1 \) на резисторе \ (R_1 \)?
    3. Найдите ток \ (I_2 \) через резистор \ (R_2 \).
    4. Какую мощность рассеивает \ (R_2 \)?
    Рисунок \ (\ PageIndex {6} \): Эти три резистора подключены к источнику напряжения так, чтобы \ (R_2 \) и \ (R_3 \) были параллельны друг другу, и эта комбинация была последовательно с \ (R_1 \).

    Стратегия

    (a) Чтобы найти эквивалентное сопротивление, сначала найдите эквивалентное сопротивление параллельного соединения \ (R_2 \) и \ (R_3 \). Затем используйте этот результат, чтобы найти эквивалентное сопротивление последовательного соединения с \ (R_1 \).

    (b) Ток через \ (R_1 \) можно найти с помощью закона Ома и приложенного напряжения. Ток через \ (R_1 \) равен току от батареи. Падение потенциала \ (V_1 \) на резисторе \ (R_1 \) (которое представляет собой сопротивление в соединительных проводах) можно найти с помощью закона Ома.{-1} = 5.10 \, \ Omega. \ Nonumber \] Общее сопротивление этой комбинации является промежуточным между значениями чистой серии и чисто параллельной (\ (20.0 \, \ Omega \) и \ (0.804 \, \ Omega \) ), соответственно).

  • Ток через \ (R_1 \) равен току, обеспечиваемому батареей: \ [I_1 = I = \ frac {V} {R_ {eq}} = \ frac {12.0 \, V} {5.10 \, \ Omega} = 2.35 \, A. \ nonumber \] Напряжение на \ (R_1 \) равно \ [V_1 = I_1R_1 = (2.35 \, A) (1 \, \ Omega) = 2.35 \, V. \ nonumber \] Напряжение, приложенное к \ (R_2 \) и \ (R_3 \), меньше напряжения, подаваемого батареей, на величину \ (V_1 \).Когда сопротивление провода велико, это может существенно повлиять на работу устройств, представленных \ (R_2 \) и \ (R_3 \).
  • Чтобы найти ток через \ (R_2 \), мы должны сначала найти приложенное к нему напряжение. Напряжение на двух параллельных резисторах одинаковое: \ [V_2 = V_3 = V — V_1 = 12.0 \, V — 2.35 \, V = 9.65 \, V. \ nonumber \] Теперь мы можем найти ток \ (I_2 \) через сопротивление \ (R_2 \) по закону Ома: \ [I_2 = \ frac {V_2} {R_2} = \ frac {9.65 \, V} {6.00 \, \ Omega} = 1.2 (6.00 \, \ Omega) = 15.5 \, W. \ nonumber \]
  • Значение

    Анализ сложных схем часто можно упростить, сведя схему к источнику напряжения и эквивалентному сопротивлению. Даже если вся схема не может быть сведена к одному источнику напряжения и одному эквивалентному сопротивлению, части схемы могут быть уменьшены, что значительно упрощает анализ.

    Упражнение \ (\ PageIndex {3} \)

    Рассмотрите электрические цепи в вашем доме.Приведите по крайней мере два примера схем, которые должны использовать комбинацию последовательных и параллельных схем для эффективной работы.

    Раствор

    Все цепи верхнего освещения параллельны и подключены к основному питанию, поэтому при перегорании одной лампочки все верхнее освещение не гаснет. У каждого верхнего света будет по крайней мере один переключатель, включенный последовательно с источником света, так что вы можете включать и выключать его.

    В холодильнике есть компрессор и лампа, которая загорается при открытии дверцы.Обычно для подключения холодильника к стене используется только один шнур. Цепь, содержащая компрессор, и цепь, содержащая цепь освещения, параллельны, но есть переключатель, включенный последовательно со светом. Термостат управляет переключателем, который включен последовательно с компрессором, чтобы контролировать температуру холодильника.

    Практическое применение

    Одним из следствий этого последнего примера является то, что сопротивление в проводах снижает ток и мощность, подаваемую на резистор.Если сопротивление провода относительно велико, как в изношенном (или очень длинном) удлинителе, то эти потери могут быть значительными. Если потребляется большой ток, падение IR в проводах также может быть значительным и может проявляться из-за тепла, выделяемого в шнуре.

    Например, когда вы роетесь в холодильнике и включается мотор, свет холодильника на мгновение гаснет. Точно так же вы можете увидеть тусклый свет в салоне, когда вы запускаете двигатель вашего автомобиля (хотя это может быть связано с сопротивлением внутри самой батареи).

    Что происходит в этих сильноточных ситуациях, показано на рисунке \ (\ PageIndex {7} \). Устройство, обозначенное символом \ (R_3 \), имеет очень низкое сопротивление, поэтому при его включении протекает большой ток. Этот увеличенный ток вызывает большее падение IR в проводах, обозначенных \ (R_1 \), уменьшая напряжение на лампочке (которое равно \ (R_2 \)), которое затем заметно гаснет.

    Рисунок \ (\ PageIndex {7} \): Почему свет тускнеет, когда включен большой прибор? Ответ заключается в том, что большой ток, потребляемый двигателем прибора, вызывает значительное падение IR в проводах и снижает напряжение на свету.

    Стратегия решения проблем: последовательные и параллельные резисторы

    1. Нарисуйте четкую принципиальную схему, обозначив все резисторы и источники напряжения. Этот шаг включает список известных значений проблемы, поскольку они отмечены на вашей принципиальной схеме.
    2. Точно определите, что необходимо определить в проблеме (определите неизвестные). Письменный список полезен.
    3. Определите, включены ли резисторы последовательно, параллельно или в комбинации последовательно и параллельно.Изучите принципиальную схему, чтобы сделать эту оценку. Резисторы включены последовательно, если через них должен последовательно проходить один и тот же ток.
    4. Используйте соответствующий список основных функций для последовательных или параллельных подключений, чтобы найти неизвестные. Есть один список для серий, а другой — для параллелей.
    5. Проверьте, являются ли ответы разумными и последовательными.

    Пример \ (\ PageIndex {4} \): объединение последовательных и параллельных цепей

    Два резистора, соединенных последовательно \ ((R_1, \, R_2) \), подключены к двум резисторам, включенным параллельно \ ((R_3, \, R_4) \).Последовательно-параллельная комбинация подключается к батарее. Каждый резистор имеет сопротивление 10,00 Ом. Провода, соединяющие резисторы и аккумулятор, имеют незначительное сопротивление. Через резистор \ (R_1 \) проходит ток 2,00 А. Какое напряжение подается от источника напряжения?

    Стратегия

    Используйте шаги предыдущей стратегии решения проблем, чтобы найти решение для этого примера.

    Раствор

    Рисунок \ (\ PageIndex {8} \): Чтобы найти неизвестное напряжение, мы должны сначала найти эквивалентное сопротивление цепи.
    1. Нарисуйте четкую принципиальную схему (Рисунок \ (\ PageIndex {8} \)).
    2. Неизвестно напряжение аккумулятора. Чтобы определить напряжение, подаваемое батареей, необходимо найти эквивалентное сопротивление.
    3. В этой схеме мы уже знаем, что резисторы \ (R_1 \) и \ (R_2 \) включены последовательно, а резисторы \ (R_3 \) и \ (R_4 \) включены параллельно. Эквивалентное сопротивление параллельной конфигурации резисторов \ (R_3 \) и \ (R_4 \) последовательно с последовательной конфигурацией резисторов \ (R_1 \) и \ (R_2 \).{-1} = 5,00 \, \ Омега. \ nonumber \] Эта параллельная комбинация включена последовательно с двумя другими резисторами, поэтому эквивалентное сопротивление схемы равно \ (R_ {eq} = R_1 + R_2 + R_ {34} = (25.00 \, \ Omega \). поэтому напряжение, подаваемое батареей, равно \ (V = IR_ {eq} = 2.00 \, A (25.00 \, \ Omega) = 50.00 \, V \).
    4. Один из способов проверить соответствие ваших результатов — это рассчитать мощность, подаваемую батареей, и мощность, рассеиваемую резисторами. Мощность, обеспечиваемая аккумулятором, равна \ (P_ {batt} = IV = 100.2R_4 \\ [4pt] & = 40.00 \, W + 40.00 \, W + 10.00 \, W + 10.00 \, W = 100. \, W. \ end {align *} \]

      Поскольку мощность, рассеиваемая резисторами, равна мощности, выделяемой батареей, наше решение кажется последовательным.

      Значение

      Если проблема имеет комбинацию последовательного и параллельного соединения, как в этом примере, ее можно уменьшить поэтапно, используя предыдущую стратегию решения проблемы и рассматривая отдельные группы последовательных или параллельных соединений.При нахождении \ (R_ {eq} \) для параллельного соединения необходимо с осторожностью относиться к обратному. Кроме того, единицы и числовые результаты должны быть разумными. Эквивалентное последовательное сопротивление должно быть больше, а эквивалентное параллельное сопротивление, например, должно быть меньше. Мощность должна быть больше для одних и тех же устройств, подключенных параллельно, по сравнению с последовательными и т. Д.

      Авторы и авторство

      • Сэмюэл Дж. Линг (Государственный университет Трумэна), Джефф Санни (Университет Лойола Мэримаунт) и Билл Мобс со многими авторами.Эта работа лицензирована OpenStax University Physics в соответствии с лицензией Creative Commons Attribution License (4.0).

      Учебное пособие по физике: Параллельные схемы

      Как упоминалось в предыдущем разделе Урока 4, два или более электрических устройства в цепи могут быть соединены последовательным или параллельным соединением. Когда все устройства соединены с использованием параллельных соединений, схема называется параллельной схемой . В параллельной схеме каждое устройство помещается в свою отдельную ветвь .Наличие ответвлений означает, что существует несколько путей, по которым заряд может проходить через внешнюю цепь. Каждый заряд, проходящий через контур внешней цепи, будет проходить через единственный резистор, присутствующий в одной ветви. Прибыв в место разветвления или узел, плата выбирает , через какую ветвь пройти на обратном пути к терминалу с низким потенциалом.

      Краткое сравнение и контраст между последовательными и параллельными цепями было сделано в предыдущем разделе Урока 4.В этом разделе подчеркивалось, что добавление большего количества резисторов в параллельную цепь приводит к довольно неожиданному результату — меньшему общему сопротивлению. Поскольку существует несколько путей, по которым может протекать заряд, добавление еще одного резистора в отдельную ветвь обеспечивает еще один путь, по которому заряд может проходить через основную область сопротивления в цепи. Это уменьшенное сопротивление в результате увеличения количества ветвей будет иметь эффект увеличения скорости, с которой течет заряд (также известной как ток).Чтобы сделать этот довольно неожиданный результат более разумным, была введена аналогия с платными дорогами. Плата за проезд — это основное место сопротивления автомобильному потоку на платной дороге. Добавление дополнительных пунктов сбора платы за проезд в пределах их собственного отделения на платной дороге обеспечит больше путей для автомобилей, проезжающих через станцию ​​взимания платы. Эти дополнительные пункты пропуска снизят общее сопротивление потоку автомобилей и увеличат скорость их движения.

      Текущий

      Скорость, с которой заряд проходит через цепь, называется током.Заряд НЕ накапливается и не начинает накапливаться в любом заданном месте, так что ток в одном месте больше, чем в других местах. Заряд НЕ расходуется резисторами таким образом, что в одном месте ток меньше, чем в другом. В параллельной схеме заряд делит на отдельные ветви, так что в одной ветви может быть больше тока, чем в другой. Тем не менее, если брать в целом, общая сумма тока во всех ветвях при сложении равна величине тока в местах за пределами ветвей.Правило, что ток везде одинаковый все еще работает, только с изюминкой. Сила тока вне ветвей равна сумме токов в отдельных ветвях. Это все еще та же величина тока, только разделенная на несколько путей.

      В форме уравнения этот принцип можно записать как

      I итого = I 1 + I 2 + I 3 + …

      , где I total — общая величина тока вне ветвей (и в батарее), а I 1 , I 2 и I 3 представляют ток в отдельных ветвях цепи.

      В этом блоке широко использовалась аналогия между расходом заряда и расходом воды. Еще раз вернемся к аналогии, чтобы проиллюстрировать, как сумма текущих значений в ветвях равна сумме вне ветвей. Поток заряда в проводах аналогичен потоку воды в трубах. Рассмотрим приведенные ниже схемы, на которых поток воды в трубах делится на отдельные ответвления. В каждом узле (место разветвления) вода проходит двумя или более отдельными путями.Скорость, с которой вода поступает в узел (измеряется в галлонах в минуту), будет равна сумме расходов в отдельных ветвях за пределами узла. Точно так же, когда две или более ветви подаются в узел, скорость, с которой вода вытекает из узла, будет равна сумме расходов в отдельных ветвях, которые подаются в узел.

      Тот же принцип разделения потока применяется к электрическим цепям. Скорость, с которой заряд поступает в узел, равна сумме расходов в отдельных ветвях за пределами узла.Это проиллюстрировано в приведенных ниже примерах. В примерах вводится новый символ схемы — буква A, заключенная в круг. Это символ амперметра — устройства, используемого для измерения силы тока в определенной точке. Амперметр способен измерять ток, оказывая при этом незначительное сопротивление потоку заряда.

      Диаграмма A показывает два резистора, подключенных параллельно с узлами в точках A и B. Заряд течет в точку A со скоростью 6 ампер и делится на два пути — один через резистор 1, а другой — через резистор 2.Ток в ветви с резистором 1 составляет 2 ампера, а ток в ветви с резистором 2 — 4 ампера. После того, как эти две ветви снова встретятся в точке B, чтобы сформировать единую линию, ток снова станет равным 6 ампер. Таким образом, мы видим, что принцип, согласно которому ток вне ветвей равен сумме тока в отдельных ветвях, справедлив.

      I всего = I 1 + I 2

      6 ампер = 2 ампера + 4 ампера

      Схема B выше может быть немного сложнее, если три резистора расположены параллельно.На схеме обозначены четыре узла, обозначенные A, B, C и D. Заряд течет в точку A со скоростью 12 ампер и делится на два пути: один проходит через резистор 1, а другой направляется к точке B (и резисторам 2). и 3). 12 ампер тока делятся на 2-амперную (через резистор 1) и 10-амперную (в направлении точки B). В точке B происходит дальнейшее разделение потока на два пути — один через резистор 2, а другой через резистор 3. Ток в 10 ампер, приближающийся к точке B, делится на 6-амперный канал (через резистор 2) и 4-канальный. -амперный тракт (через резистор 3).Таким образом, видно, что значения тока в трех ветвях составляют 2 ампера, 6 ампер и 4 ампера, и что сумма значений тока в отдельных ветвях равна току вне ветвей.

      I всего = I 1 + I 2 + I 3

      12 ампер = 2 ампер + 6 ампер + 4 ампер

      Анализ потока в точках C и D также может быть проведен, и будет замечено, что сумма расходов потока в этих точках равна скорости потока, находящейся непосредственно за этими точками.

      Эквивалентное сопротивление

      Фактическая величина тока всегда изменяется обратно пропорционально величине общего сопротивления. Существует четкая взаимосвязь между сопротивлением отдельных резисторов и общим сопротивлением набора резисторов. Чтобы исследовать эту взаимосвязь, давайте начнем с простейшего случая, когда два резистора помещены в параллельные ветви, каждый из которых имеет одинаковое значение сопротивления 4 Ом.Поскольку схема предлагает два равных путей для потока заряда, только половина заряда выберет для прохождения через данную ветвь. В то время как каждая отдельная ветвь предлагает сопротивление 4 Ом любому заряду, который проходит через нее, только половина всего заряда, протекающего по цепи, будет встречать сопротивление 4 Ом этой отдельной ветви. Таким образом, что касается батареи, которая накачивает заряд, наличие двух параллельно подключенных резисторов 4 Ом было бы эквивалентно наличию одного резистора 2 Ом в цепи.Таким же образом, наличие двух параллельно подключенных резисторов сопротивлением 6 Ом было бы эквивалентно наличию в цепи одного резистора сопротивлением 3 Ом. А наличие двух параллельных резисторов 12 Ом было бы эквивалентно наличию в цепи одного резистора 6 Ом.

      Теперь давайте рассмотрим другой простой случай, когда параллельно соединены три резистора, каждый из которых имеет одинаковое сопротивление 6 Ом. При трех равных путях прохождения заряда через внешнюю цепь только одна треть заряда будет проходить через данную ветвь.Каждая отдельная ветвь обеспечивает сопротивление 6 Ом проходящему через нее заряду. Однако тот факт, что только одна треть заряда проходит через определенную ветвь, означает, что общее сопротивление цепи составляет 2 Ом. Что касается батареи, которая накачивает заряд, наличие трех параллельных резисторов 6 Ом было бы эквивалентно наличию одного резистора 2 Ом в цепи. Таким же образом, наличие трех параллельно подключенных резисторов сопротивлением 9 Ом было бы эквивалентно наличию в цепи одного резистора сопротивлением 3 Ом.А наличие трех параллельных резисторов 12 Ом было бы эквивалентно наличию одного резистора 4 Ом в цепи.

      Это концепция эквивалентного сопротивления. Эквивалентное сопротивление схемы — это величина сопротивления, которая потребуется одному резистору, чтобы сравняться с общим эффектом от набора резисторов, присутствующих в схеме. Для параллельных цепей математическая формула для вычисления эквивалентного сопротивления (R eq ):

      1 / R экв. = 1 / R 1 + 1 / R 2 + 1 / R 3 +…

      , где R 1 , R 2 и R 3 — значения сопротивления отдельных резисторов, подключенных параллельно. Приведенные выше примеры можно рассматривать как простые случаи, в которых все пути обладают одинаковым сопротивлением отдельному заряду, который проходит через них. Приведенные выше простые случаи были выполнены без использования уравнения. Тем не менее, уравнение подходит как для простых случаев, когда резисторы ответвления имеют одинаковые значения сопротивления, так и для более сложных случаев, когда резисторы ответвления имеют разные значения сопротивления.Например, рассмотрим применение уравнения к одному простому и одному сложному случаю ниже.

      Случай 1 : Три резистора 12 Ом включены параллельно

      1 / R экв = 1 / R 1 + 1 / R 2 + 1 / R 3

      1 / R экв = 1 / (12 Ом) + 1 / (12 Ом) + 1 / (12 Ом)

      Использование калькулятора …

      1 / R экв. = 0,25 Ом -1

      R экв = 1 / (0,25 Ом -1 )

      R экв = 4,0 Ом

      Случай 2 : резисторы 5,0 Ом, 7,0 Ом и 12 Ом подключены параллельно

      1 / R экв = 1 / R 1 + 1 / R 2 + 1 / R 3

      1 / R экв = 1 / (5.0 Ом) + 1 / (7,0 Ом) + 1 / (12 Ом)

      Использование калькулятора …

      1 / R экв. = 0,42619 Ом-1

      R экв = 1 / (0,42619 Ом -1 )

      R экв = 2,3 Ом


      Ваша очередь попробовать

      Нужно больше практики? Используйте два параллельных резистора виджет ниже, чтобы попробовать некоторые дополнительные проблемы.Введите любые два желаемых значения сопротивления. Используйте свой калькулятор, чтобы определить значения R eq . Затем нажмите кнопку Отправить , чтобы проверить свои ответы. Попробуйте столько раз, сколько хотите, с разными значениями сопротивления.

      Падения напряжения для параллельных ветвей

      В разделе «Схемы» учебного пособия «Физический класс» подчеркивалось, что любое повышение напряжения, полученное за счет заряда в батарее, теряется из-за заряда, когда он проходит через резисторы внешней цепи.Общее падение напряжения во внешней цепи равно увеличению напряжения при прохождении заряда через внутреннюю цепь. В параллельной схеме заряд не проходит через каждый резистор; скорее, он проходит через единственный резистор. Таким образом, полное падение напряжения на этом резисторе должно соответствовать напряжению батареи. Не имеет значения, проходит ли заряд через резистор 1, резистор 2 или резистор 3, падение напряжения на резисторе, которое выбирает для прохождения , должно равняться напряжению батареи.В форме уравнения этот принцип может быть выражен как

      В батарея = В 1 = В 2 = В 3 = …

      Если три резистора размещены в параллельных ветвях и питаются от 12-вольтовой батареи, то падение напряжения на каждом из трех резисторов составляет 12 вольт. Заряд, протекающий по цепи, встретит только один из этих трех резисторов и, таким образом, столкнется с одним падением напряжения на 12 вольт.

      Диаграммы электрических потенциалов

      были представлены в Уроке 1 этого устройства и впоследствии использовались для иллюстрации последовательных падений напряжения, происходящих в последовательных цепях.Диаграмма электрических потенциалов — это концептуальный инструмент для представления разности электрических потенциалов между несколькими точками электрической цепи. Рассмотрим приведенную ниже принципиальную схему и соответствующую диаграмму электрических потенциалов.

      Как показано на диаграмме электрического потенциала, все позиции A, B, C, E и G имеют высокий электрический потенциал. Один заряд выбирает только один из трех возможных путей; таким образом, в позиции B один заряд будет двигаться к точкам C, E или G, а затем пройдет через резистор, находящийся в этой ветви.Заряд не теряет свой высокий потенциал до тех пор, пока он не пройдет через резистор, либо от C к D, от E к F или от G к H. После того, как он пройдет через резистор, заряд вернется почти до 0 вольт и вернется к отрицательному значению. клемму аккумуляторной батареи для повышения ее напряжения. В отличие от последовательных цепей, заряд в параллельной цепи встречает единственное падение напряжения на своем пути через внешнюю цепь.

      Ток через данную ветвь можно предсказать, используя уравнение закона Ома, падение напряжения на резисторе и сопротивление резистора.Поскольку падение напряжения на каждом резисторе одинаково, фактором, определяющим, что резистор имеет наибольший ток, является сопротивление. Резистор с наибольшим сопротивлением испытывает наименьший ток, а резистор с наименьшим сопротивлением — наибольший ток. В этом смысле можно сказать, что заряд (как и люди) выбирает путь наименьшего сопротивления. В форме уравнения это может быть указано как

      I 1 = Δ V 1 / R 1 I 2 = Δ V 2 / R 2 I 3 = Δ V 3 / R 3

      Этот принцип иллюстрируется схемой, показанной ниже.Произведение I • R одинаково для каждого резистора (и равно напряжению батареи). Тем не менее, ток у каждого резистора разный. Ток наибольший там, где сопротивление наименьшее, и ток наименьший, где сопротивление наибольшее.

      Математический анализ параллельных цепей

      Приведенные выше принципы и формулы могут использоваться для анализа параллельной цепи и определения значений тока и разности электрических потенциалов на каждом из резисторов в параллельной цепи.Их использование будет продемонстрировано математическим анализом схемы, показанной ниже. Цель состоит в том, чтобы использовать формулы для определения эквивалентного сопротивления цепи (R eq ), тока через батарею (I ), а также падений напряжения и тока для каждого из трех резисторов.

      Анализ начинается с использования значений сопротивления отдельных резисторов для определения эквивалентного сопротивления цепи.

      1 / R экв = 1 / R 1 + 1 / R 2 + 1 / R 3 = (1/17 Ω) + (1/12 Ω) + (1/11 Ω)

      1 / R экв = 0.23306 Ом -1

      R экв = 1 / (0,23306 Ом -1 )

      R экв = 4,29 Ом

      (округлено от 4,29063 Ом)

      Теперь, когда известно эквивалентное сопротивление, ток в батарее можно определить с помощью уравнения закона Ома. При использовании уравнения закона Ома (ΔV = I • R) для определения тока в батарее важно использовать напряжение батареи для ΔV и эквивалентное сопротивление для R.Расчет показан здесь:

      I tot = ΔV аккумулятор / R eq = (60 В) / (4,29063 Ом)

      I до = 14,0 А

      (округлено от 13,98396 А)

      Напряжение батареи 60 В представляет собой усиление электрического потенциала за счет заряда, проходящего через батарею. Заряд теряет такое же количество электрического потенциала при любом прохождении через внешнюю цепь.То есть падение напряжения на каждом из трех резисторов такое же, как и напряжение, полученное в батарее:

      ΔV аккумулятор = ΔV 1 = ΔV 2 = ΔV 3 = 60 В

      Осталось определить три значения — ток в каждом отдельном резисторе. Закон Ома снова используется для определения значений тока для каждого резистора — это просто падение напряжения на каждом резисторе (60 В), деленное на сопротивление каждого резистора (указанное в формулировке задачи).Расчеты показаны ниже.

      I 1 = ΔV 1 / R 1

      I 1 = (60 В) / (17 Ом)

      I 1 = 3,53 А

      I 2 = ΔV 2 / R 2

      I 2 = (60 В) / (12 Ом)

      I 2 = 5,00 А

      I 3 = ΔV 3 / R 3

      I 3 = (60 В) / (11 Ом)

      I 3 = 5.45 А

      В качестве проверки точности выполненных математических расчетов целесообразно проверить, удовлетворяют ли вычисленные значения принципу, согласно которому сумма значений тока для каждого отдельного резистора равна общему току в цепи (или в батарее). . Другими словами, I tot = I 1 + I 2 + I 3 ?

      Является ли I tot = I 1 + I 2 + I 3 ?

      Is 14.0 ампер = 3,53 ампер + 5,00 ампер + 5,45 ампер?

      14,0 А = 13,98 А?

      Да !!

      (Разница в 0,02 ампера — это просто результат предыдущего округления значения I до с 13,98.)

      Математический анализ этой параллельной цепи включал смесь концепций и уравнений. Как это часто бывает в физике, отделение понятий от уравнений при принятии решения физической проблемы является опасным актом.Здесь необходимо учитывать концепции, согласно которым падение напряжения на каждом из трех резисторов равно напряжению батареи и что сумма тока в каждом резисторе равна общему току. Эти представления необходимы для завершения математического анализа. В следующей части Урока 4 будут исследованы комбинированные или составные схемы, в которых одни устройства включены параллельно, а другие — последовательно.

      Создавайте, решайте и проверяйте свои собственные проблемы, используя виджет Equivalent Resistance ниже.Создайте себе проблему с любым количеством резисторов и любыми номиналами. Решать проблему; затем нажмите кнопку «Отправить», чтобы проверить свой ответ.

      Мы хотели бы предложить … Зачем просто читать об этом и когда можно с этим взаимодействовать? Взаимодействовать — это именно то, что вы делаете, когда используете одно из интерактивных материалов The Physics Classroom. Мы хотели бы предложить вам совместить чтение этой страницы с использованием нашего интерактивного средства построения цепей постоянного тока.Вы можете найти его в разделе Physics Interactives на нашем сайте. Построитель цепей постоянного тока предоставляет учащемуся набор для построения виртуальных цепей. Вы можете легко перетащить источники напряжения, резисторы и провода на рабочее место, а также расположить и подключить их так, как захотите. Вольтметры и амперметры позволяют измерять падение тока и напряжения. Нажатие на резистор или источник напряжения позволяет изменять сопротивление или входное напряжение. Это просто. Это весело. И это безопасно (если вы не используете его в ванне).


      Проверьте свое понимание

      1. По мере того, как в цепь добавляется все больше и больше резисторов, эквивалентное сопротивление цепи ____________ (увеличивается, уменьшается) и общий ток цепи ____________ (увеличивается, уменьшается).

      2.Три одинаковых лампочки подключены к D-ячейке, как показано ниже. P, Q, X, Y и Z обозначают местоположения вдоль цепи. Какое из следующих утверждений верно?

      а. Ток в точке Y больше, чем ток в точке Q.

      г. Ток на Y больше, чем на P.

      г. Ток в точке Y больше, чем ток в точке Z.

      г. Ток в точке P больше, чем ток в точке Q.

      e.Ток на Q больше, чем на P.

      ф. Сила тока одинакова во всех местах.

      3. Три одинаковые лампочки подключены к D-ячейке, как показано ниже. P, Q, X, Y и Z обозначают местоположения вдоль цепи. В каком (их) месте (ах) ток будет …

      а. … так же, как у X?

      г…. такой же, как у Q?

      г. … так же, как у Y?

      г. … меньше, чем у Q?

      e. … меньше, чем у P?

      ф. … вдвое больше, чем у Z?

      г. … в три раза больше, чем в Y?

      4. Какие настройки можно внести в схему ниже, чтобы уменьшить ток в ячейке? Перечислите все подходящие варианты.

      а. Увеличьте сопротивление лампы X.

      г. Уменьшите сопротивление лампы X.

      г. Увеличьте сопротивление лампы Z.

      г. Уменьшите сопротивление лампы Z.

      .

      e. Увеличьте напряжение ячейки (как-нибудь).

      ф. Уменьшите напряжение ячейки (как-нибудь).

      г. Снимите лампу Y.

      .

      5.Аккумулятор на 12 В, резистор на 12 Ом и резистор на 4 Ом подключаются, как показано на рисунке. Ток в резисторе 12 Ом равен ____ току в резисторе 4 Ом.

      а. 1/3

      г. 1/2

      г. 2/3

      г. то же, что

      e.1,5 раза

      ф. дважды

      г. трижды

      ч. четыре раза


      6. Аккумулятор на 12 В, резистор на 12 Ом и резистор на 4 Ом подключаются, как показано.Падение напряжения на резисторе 12 Ом равно ____ падению напряжения на резисторе 4 Ом.

      а. 1/3

      г. 1/2

      г. 2/3

      г. то же, что

      e. 1,5 раза

      ф.дважды

      г. трижды

      ч. четыре раза

      7. Аккумулятор на 12 В и резистор на 12 Ом подключаются, как показано на схеме. Резистор на 6 Ом добавлен к резистору на 12 Ом, чтобы создать цепь Y, как показано.Падение напряжения на резисторе 6 Ом в цепи Y равно ____ падению напряжения на резисторе в цепи X.

      а. больше, чем

      г. меньше

      г. то же, что

      8. Используйте свое понимание эквивалентного сопротивления, чтобы заполнить следующие утверждения:

      а. Два резистора сопротивлением 6 Ом, помещенные параллельно, обеспечат сопротивление, эквивалентное сопротивлению одного резистора _____ Ом.

      г. Три резистора 6 Ом, помещенные параллельно, обеспечат сопротивление, эквивалентное одному резистору _____ Ом.

      г. Три резистора сопротивлением 8 Ом, помещенные параллельно, обеспечат сопротивление, эквивалентное сопротивлению одного резистора _____ Ом.

      г. Три резистора с сопротивлением 2 Ом, 4 Ом и 6 Ом размещены параллельно. Они обеспечили бы сопротивление, эквивалентное одному резистору _____ Ом.

      e. Три резистора с сопротивлением 5 Ом, 6 Ом и 7 Ом размещены параллельно.Они обеспечили бы сопротивление, эквивалентное одному резистору _____ Ом.

      ф. Три резистора с сопротивлением 12 Ом, 6 Ом и 21 Ом размещены параллельно. Они обеспечили бы сопротивление, эквивалентное одному резистору _____ Ом.

      9. На основании ваших ответов на вышеуказанный вопрос заполните следующую формулировку:

      Общее или эквивалентное сопротивление трех параллельно включенных резисторов будет _____.

      а. больше, чем сопротивление самого большого значения R.

      г. меньше, чем сопротивление наименьшего значения R из трех.

      г. где-то между наименьшим значением R и наибольшим значением R.

      г. … бред какой то! Такого обобщения сделать нельзя. Результаты меняются.

      10. Три резистора включены параллельно.При размещении в цепи с источником питания 12 В. Определите эквивалентное сопротивление, полный ток цепи, падение напряжения и ток в каждом резисторе.


      Параллельные схемы

      Ваш браузер не поддерживает Java-апплеты

      Схема с более чем одним Путь прохождения тока представляет собой параллельную цепь.

      НАПРЯЖЕНИЕ В ПАРАЛЛЕЛЬНЫХ ЦЕПЯХ

      Общее напряжение равно напряжение любого параллельного сопротивления.

      ТОК В ПАРАЛЛЕЛЬНЫХ ЦЕПЯХ

      Полный ток равен сумма тока каждого параллельного компонента.

      ПАРАЛЛЕЛЬНОЕ СОПРОТИВЛЕНИЕ ЦЕПИ

      Общее сопротивление может быть рассчитывается по закону Ома, если известны напряжение и полный ток.

      Общее сопротивление всегда меньше наименьшего значения сопротивления.

      Метод равных значений

      Для параллельных сопротивлений в какие все резисторы имеют одинаковое значение, сопротивление можно рассчитать по формуле разделив номинал одного из резисторов на количество резисторов.

      Взаимный метод

      Для параллельных сопротивлений в какие все резисторы имеют одинаковое значение, сопротивление можно рассчитать по формуле разделив номинал одного из резисторов на количество резисторов.

      1 / R T = 1 / R 1 + 1 / R 2 + 1 / R N

      R EQ = 1 / (1 / R 1 + 1 / R 2 + … + 1 / R N )

      Метод произведения на сумму

      Для расчета сопротивления двух параллельных резисторов можно использовать эту формулу: 90 · 109

      R EQ = (R 1 * R 2 ) / (R 1 + R 2 )

      Правило приближения 10 к 1

      Если подключены два резистора параллельно, и один резистор в 10 или более раз больше по стоимости, чем другой резистор, резистор большего номинала можно не учитывать.

      ПРОВОДИМОСТЬ

      Общая проводимость равна сумме проводимости каждого компонента.

      ПИТАНИЕ В ПАРАЛЛЕЛЬНЫХ ЦЕПЯХ

      Суммарная мощность равна сумма мощности каждого компонента. (Это то же самое, что и с серией схемы).

      Правила для параллельных цепей постоянного тока

      1. Такое же напряжение существует через каждую ветвь параллельной цепи и равно напряжению источника.
      2. Ток через параллельная ветвь обратно пропорциональна величине сопротивления ветка.
      3. Полный ток параллельная цепь равна сумме отдельных токов ответвления цепь
      4. Эквивалентное сопротивление параллельная цепь находится по общему уравнению Req = 1 / (1 / R1 + 1 / R2 + 1 / Rn)
      5. Общая мощность, потребляемая в параллельная схема равна сумме мощности, потребляемой индивидуумом резисторы.

      ПАРАЛЛЕЛЬНЫЙ АНАЛИЗ ЦЕПЕЙ

      1. Соблюдайте принципиальную схему внимательно или при необходимости нарисуйте.
      2. Обратите внимание на указанные значения и значения, которые необходимо найти.
      3. Выберите подходящий уравнения, которые будут использоваться при решении для неизвестных величин на основе известных количества.
      4. Подставьте известные значения в уравнении, которое вы выбрали, и решите для неизвестного значения.

      ПАРАЛЛЕЛЬНОЕ УСТРАНЕНИЕ НЕПОЛАДОК ЦЕПИ

      Когда в ветви параллельной сети сопротивление ветви увеличивается и общее сопротивление цепи увеличивается. Это вызывает уменьшение общего Текущий.

      Короткое замыкание всегда приводит в отсутствии тока, протекающего через другие ветви цепи.

      Сопротивление в параллельной цепи

      Сопротивление в параллельной цепи
      На примерной схеме, рис. 3-44, два резистора подключены параллельно через 5-вольтовую батарею.Каждый имеет значение сопротивления 10 Ом. Формируется полная цепь, состоящая из двух параллельных путей, и ток течет, как показано.

      Рисунок 3-44. — Два равных резистора, включенных параллельно.

      Вычисление отдельных токов показывает, что через каждое сопротивление проходит половина ампера. Полный ток, протекающий от батареи к переходу резисторов и возвращающийся от резисторов к батарее, равен 1 амперам.

      Общее сопротивление цепи можно рассчитать, используя
      значения полного напряжения (E T ) и полного тока (I T ).

      ПРИМЕЧАНИЕ. С этого момента в примерах задач будут использоваться сокращения и символы для электрических величин.

      Дано:


      Решение:


      Это вычисление показывает, что полное сопротивление составляет 5 Ом; половина номинала любого из двух резисторов.

      Поскольку полное сопротивление параллельной цепи меньше, чем у любого из отдельных резисторов, полное сопротивление параллельной цепи не является суммой значений отдельных резисторов, как это было в случае последовательной цепи.Общее сопротивление параллельно включенных резисторов также называется ЭКВИВАЛЕНТНЫМ СОПРОТИВЛЕНИЕМ. (R экв. ). Термины полное сопротивление и эквивалентное сопротивление используются как синонимы.

      Существует несколько методов определения эквивалентного сопротивления параллельных цепей. Лучший метод для данной схемы зависит от количества и номинала резисторов. Для схемы, описанной выше, где все резисторы имеют одинаковое значение, используется следующее простое уравнение:


      Это уравнение действительно для любого количества параллельных резисторов с РАВНЫМ ЗНАЧЕНИЕМ.

      Пример. Параллельно подключены четыре резистора на 40 Ом. Какое у них эквивалентное сопротивление?

      Дано:


      Решение:


      На рис. 3-45 показаны два резистора разного номинала, включенные параллельно. Поскольку показан полный ток, можно рассчитать эквивалентное сопротивление.

      Рисунок 3-45. — Пример схемы с неравнопараллельными резисторами.


      Дано:


      Решение:


      Эквивалентное сопротивление цепи, показанной на рисунке 3-45, меньше, чем у любого из двух резисторов (R 1 , R 2 ).Важно помнить, что эквивалентное сопротивление параллельной цепи всегда меньше, чем сопротивление любой ветви.

      Эквивалентное сопротивление можно найти, зная отдельные значения сопротивления и напряжение источника. Вычисляя ток каждой ветви, складывая токи ветви для вычисления общего тока и разделив напряжение источника на общий ток, можно найти общий ток. Этот метод хоть и эффективен, но довольно длительный. Более быстрый способ найти эквивалентное сопротивление — использовать общую формулу для параллельных резисторов:


      Если вы примените общую формулу к схеме, показанной на рисунке 3-45, вы получите такое же значение для эквивалентного сопротивления (2 Ом), что и было полученный в предыдущем расчете, в котором использовались напряжение источника и полный ток.

      Дано:


      Решение:


      Преобразуйте дроби к общему знаменателю.


      Поскольку обе стороны взаимны (разделены на одну), не обращайте внимания на обратную функцию.


      Формула, которую вы дали для одинаковых резисторов, включенных параллельно


      , является упрощением общей формулы для параллельных резисторов


      Существуют и другие упрощения общей формулы для параллельных резисторов, которые можно использовать для расчета общей суммы или эквивалентное сопротивление в параллельной цепи.

      ВЗАИМНЫЙ МЕТОД. — Этот метод основан на взятии обратной величины для каждой стороны уравнения. Это представляет собой общую формулу для резисторов, включенных параллельно:


      Эта формула используется для определения эквивалентного сопротивления ряда неравных параллельных резисторов. При решении этих задач вы должны найти наименьший общий знаменатель. Если вы не знаете, как найти наименьший общий знаменатель, освежите его в математике, том 1, NAVEDTRA 10069 (серия).

      Пример: три резистора подключены параллельно, как показано на рисунке 3-46. Значения резистора: R 1 = 20 Ом, R 2 = 30 Ом, R 3 = 40 Ом. Какое эквивалентное сопротивление? (Используйте обратный метод.)

      Рисунок 3-46. — Пример параллельной схемы с неравными резисторами ответвления.


      Дано:


      Решение:

      ПРОДУКТ ПРЕВЫШАЕТ СУММУ. — Удобный способ найти эквивалентное или общее сопротивление двух параллельных резисторов — использовать следующую формулу.


      Это уравнение, называемое формулой произведения на сумму, используется так часто, что его следует сохранить в памяти.

      Пример. Какое эквивалентное сопротивление резисторов на 20 Ом и 30 Ом, подключенных параллельно, как показано на рисунке 3-47?

      Рисунок 3-47. — Параллельная схема с двумя неравными резисторами.

      Дано:


      Решение:


      Четыре равных резистора подключены параллельно, каждый резистор имеет омическое значение 100 Ом, каково эквивалентное сопротивление?

      Три резистора, соединенных параллельно, имеют номиналы 12 кОм, 20 кОм и 30 кОм.Какое эквивалентное сопротивление?

      Два резистора, соединенных параллельно, имеют номиналы 10 кОм и 30 кОм. Какое эквивалентное сопротивление?

      Резисторы

      в схемах — Практика — Физический гипертекст

      Давайте начнем процесс с объединения резисторов. В этой схеме четыре последовательных пары.

      слева
      R с = 3 Ом + 1 Ом
      R с = 4 Ом
      R с = 4 Ом + 2 Ом
      R с = 6 Ом
      правый
      R с = 2 Ом + 3 Ом
      R с = 5 Ом
      R с = 1 Ом + 4 Ом
      R s = 5 Ом

      Эти пары образуют две параллельные цепи, одну слева и одну справа.

      слева
      1 = 1 + 1
      R p 4 Ом 6 Ом
      R p = 12 Ом = 2.4 Ом
      5
      правый
      1 = 1 + 1
      R p 5 Ом 5 Ом
      R p = 5 Ом = 2.5 Ом
      2

      Каждый набор из четырех резисторов включен последовательно с другим.

      слева
      R с = 2,4 Ом + 0,6 Ом
      R с = 3 Ом
      правый
      R с = 2,5 Ом + 0,5 Ом
      R с = 3 Ом

      Левая и правая половины цепи параллельны друг другу и батарее.

      1 = 1 + 1 = 2
      R p 3 Ом 3 Ом 3 Ом
      R p = 3 Ом = 1.5 Ом
      2

      Теперь, когда у нас есть эффективное сопротивление всей цепи, давайте определим ток источника питания по закону Ома.

      I итого = V итого + 24 В = 16 А
      R Итого 1.5 Ом

      А теперь пройдемся по цепи (не в буквальном смысле конечно). На каждом соединении ток будет делиться: больше по пути с меньшим сопротивлением и меньше по пути с большим сопротивлением. Поскольку заряд не протекает нигде в полной цепи, ток будет одинаковым для всех элементов, последовательно соединенных друг с другом.

      Левая и правая половины цепи идентичны по общему сопротивлению, что означает, что ток будет равномерно делиться между ними.


      8 A для резистора 0,6 Ом
      на слева .

      8 A для резистора 0,5 Ом
      на правой стороне .

      С каждой стороны ток снова делится на две параллельные ветви.

      Ветви слева имеют сопротивления в соотношении…
      R 1 и 3 = 4 Ом + 2
      R 2 и 4 6 Ом 3

      что означает, что токи разделятся в соотношении…
      для резисторов 1 Ом и 3 Ом
      на слева .
      для резисторов 2 Ом и 4 Ом
      на слева .
      Ветви справа идентичны, поэтому ток разделяется на две равные половины.
      для резисторов 2 Ом и 3 Ом
      на правой стороне .
      для резисторов 1 Ом и 4 Ом
      на правой стороне .

      резисторов, включенных последовательно и параллельно | Комбинации резисторов

      Результаты обучения

      • Рассчитайте общее сопротивление различных комбинаций резисторов, т. Е. Последовательных, параллельных и последовательно-параллельных.
      • Покажите, как резисторы используются в качестве делителей напряжения и тока.
      • Рассчитайте сопротивление и значение мощности для последовательного резистора, понижающего напряжение.

      Отдельные резисторы могут быть соединены вместе последовательно, параллельно или в комбинации последовательно и параллельно. Это приводит к более сложной схеме, полное сопротивление которой представляет собой комбинацию отдельных резисторов.

      Комбинация резисторов серии

      Для подключения резисторов серии , они соединяются конец к концу вместе в одну линию, как показано для Рисунок 1 .Характеристики последовательно соединенных резисторов можно резюмировать следующим образом:

      • Общее сопротивление цепи ( R T ) увеличивается, если дополнительные резисторы подключаются последовательно, и уменьшается, если резисторы удалены.
      • Чтобы определить общее сопротивление цепи, просто найдите сумму отдельных сопротивлений нагрузок.
      • В этом примере, если резисторы имеют маркировку R 1 , R 2 и R 3 , то общее сопротивление R T рассчитывается по формуле

      Рисунок 1 Резисторы, подключенные последовательно.

      ПРИМЕР 1

      Проблема: Три резистора, R 1 (4 Ом), R 2 (50 Ом) и R 3 (75 Ом) подключены последовательно, как показано на рис. 2 . Определите значение общего сопротивления комбинированной цепи.

      Рисунок 2 Схема для примера 1.

      Решение:

      Резисторы, соединенные последовательно, используются как делители напряжения , , как показано на схеме , Рисунок 3 .Делители напряжения широко используются в схемах, где один источник напряжения должен обеспечивать несколько разных значений напряжения для разных частей схемы.

      Характеристики схемы последовательного делителя напряжения можно резюмировать следующим образом:

      • Через каждый резистор протекает одинаковое количество тока.
      • Входное напряжение делится пропорционально между последовательно соединенными резисторами.
      • Падение напряжения на резисторе в последовательной цепи прямо пропорционально сопротивлению резистора.
      • Чем выше значение сопротивления, тем больше падение напряжения.

      Рисунок 3 Схема делителя напряжения.

      Для схемы делителя напряжения падение напряжения на каждом резисторе обычно является фактором, который необходимо определить. Падение напряжения на любом резисторе пропорционально отношению его сопротивления к общему сопротивлению цепи.

      Формула делителя напряжения позволяет рассчитать падение напряжения на любом из последовательно подключенных резисторов без необходимости сначала рассчитывать значение тока цепи.Заявлено в виде формулы:

      ПРИМЕР 2

      Резисторы R 1 (5 кОм), R 2 (3 кОм) и R 3 90 ) соединены последовательно, образуя делитель напряжения, как показано на рис. 4 . Если на схему подается входное напряжение 9 вольт, рассчитайте значение падения напряжения на каждом из резисторов, используя формулу делителя напряжения.

      Рисунок 4 Схема для примера 2.

      Решение:

      ПРИМЕР 3

      У вас есть источник на 120 В и вы хотите использовать последовательно понижающий резистор в сочетании с контрольной лампой 6 В при 150 мА для индикации подачи питания ( Рисунок 5 ). Определите значение сопротивления падения серии и требуемую мощность.

      Рисунок 5 Схема для примера 3.

      Решение:

      Параллельная комбинация резисторов

      Резисторы подключаются параллельно , соединяя их параллельно как показано на рис. 6 .Обратите внимание, что два конца резисторов подключены к одним и тем же двум точкам.

      Характеристики параллельно соединенных резисторов можно резюмировать следующим образом:

      • Общее сопротивление (R T ) сформированной цепи на меньше, чем на , чем у самого низкого значения сопротивления, присутствующего в любой из ветвей.
      • Каждый резистор обеспечивает отдельный параллельный путь для прохождения тока.
      • Если у вас есть несколько резисторов одинакового номинала, подключенных параллельно, то общее сопротивление легче всего найти, разделив общее значение сопротивления на количество подключенных резисторов.Для трех параллельно подключенных резисторов по 150 Ом общее сопротивление составляет

      Рисунок 6 Сопротивление параллельно.

      Чтобы найти полное сопротивление двух неравных значений резисторов, соединенных параллельно (очень распространенное использование), используется формула произведения на сумму. Эта формула:

      ПРИМЕР 4

      Проблема: Резистор 60 Ом подключается параллельно резистору 40 Ом, как показано на рис. 7 .Определите значение общего комбинированного сопротивления двух компонентов, используя формулу «произведение на сумму».

      Рисунок 7 Схема для примера 4.

      Решение:

      Формула произведения на сумму лучше всего работает для двух резисторов, включенных параллельно. Если параллельно подключено более двух резисторов, использование становится труднее и менее практичным. Для более чем двух резисторов разных номиналов, соединенных параллельно, используется общая формула для полного сопротивления параллельной цепи.Эта формула:

      ПРИМЕР 5

      Проблема: Три резистора, R 1 (120 Ом), R 2 (60 Ом) и R 3 3 3 40 Ом) подключены параллельно, как показано на Рисунок 8 . Определите значение общего сопротивления комбинированной цепи.

      Рисунок 8 Схема для примера 5.

      Решение:

      Параллельные резистивные цепи можно рассматривать как делители тока , потому что ток разделяется или делится между различными резисторами, как показано на рисунке 9 .

      Характеристики схемы параллельного делителя тока можно резюмировать следующим образом:

      • Ток, протекающий через каждый резистор ответвления, обратно пропорционален его значению сопротивления.
      • Чем меньше значение сопротивления, тем больше ток, и наоборот.
      • Резисторы с одинаковым сопротивлением будут пропускать через них одинаковый ток.
      • Формула, описывающая делитель тока, аналогична формуле для делителя напряжения и может быть выражена следующим образом:

      Рис. 9 Схема делителя тока.

      ПРИМЕР 6

      Проблема: Резисторы R 1 R 2 и R 3 (2 Ом, 3 Ом и 6 Ом соответственно) подключены параллельно, как показано на Рисунок 10 . Используйте формулу делителя тока, чтобы вычислить значение тока, протекающего через каждый из нагрузочных резисторов, если общий ток, протекающий в цепи, составляет 10 ампер.

      Рисунок 10 Схема для примера 6.

      Решение:

      ПРИМЕР 7

      Проблема: При параллельном подключении дополнительных нагрузок общее сопротивление цепи уменьшается. Для цепи, показанной на рис. 11 , , определите полное сопротивление цепи при каждом из следующих рабочих условий:

      1. Переключатели 1 и 2 замкнуты.
      2. Переключатели 1, 2 и 3 замкнуты.

      Рисунок 11 Схема для примера 7.

      Решение:

      Последовательно-параллельная комбинация резисторов

      Комбинированные резистивные цепи, также известные как последовательно-параллельные резистивные цепи , объединяют резисторы последовательно с резисторами, включенными параллельно, как показано на рисунке 12 .

      Правила, регулирующие эти цепи, такие же, как и правила, разработанные для последовательных и параллельных цепей. Сначала определяется сопротивление совокупного полного сопротивления параллельной части.Затем общее сопротивление параллельной части добавляется к любому последовательному сопротивлению, чтобы найти общее сопротивление последовательно-параллельной комбинированной цепи.

      Рисунок 12 Последовательно-параллельное соединение резисторов.

      ПРИМЕР 8

      Проблема: Резистор 9 Ом, R 1 , и резистор 60 Ом, R 2 , подключены параллельно друг другу и последовательно с резистор 40 Ом, R 3 , как показано на Рис.13 .Определите общее сопротивление этой последовательно-параллельной комбинации резисторов.

      Рисунок 13 Схема для примера 8.

      Решение:

      ПРИМЕР 9

      Проблема: Показания сопротивления можно использовать для проверки схем на наличие неисправностей. Как определено в предыдущем примере, нормальное полное сопротивление этой последовательно-параллельной схемы , рис. 14, составляет 60 Ом.

      1. Найдите новое значение R T , если произойдет отказ резистора R 1 открыт , в то время как значения сопротивления R 2 и R 3 останутся одно и тоже.
      2. Аналогичным образом найдите новое значение R T , если резистор R 3 будет поврежден закорочен , а значения сопротивления R 1 и R 2 останутся тоже самое.

      Рисунок 14 Цепь для примера 9.

      Решение:

      1. При разомкнутом отказе R 1 цепь будет состоять из R 3 последовательно с R 2 и всего сопротивление будет:

      1. При коротком замыкании R 3 цепь будет состоять из R 1 параллельно с R 2 , а общее сопротивление будет:

      Вопросы для обзора

      1. Рассчитайте общее сопротивление для каждой из следующих цепей резисторов:
        1. Последовательная цепь: R1 = 40 Ом, R2 = 75 Ом
        2. Параллельная цепь: R1 = 200 Ом, R2 = 200 Ом, R3 = 200 Ом
        3. Последовательная цепь: R1 = 2000 Ом, R2 = 6000 Ом, R3 = 2200 Ом
        4. Параллельная цепь: R1 = 14 Ом, R2 = 32 Ом
        5. Последовательная цепь: R1 = 4700 Ом, R2 = 800 Ом, R3 = 200 Ом
        6. Параллельная цепь: R1 = 60 Ом, R2 = 30 Ом, R3 = 15 Ом
      2. 9 0117 Резисторы R 1 , R 2 и R 3 (50 Ом, 30 Ом и 20 Ом соответственно) подключены последовательно через приложенное напряжение 200 В, чтобы сформировать делитель напряжения.Используя формулу делителя напряжения, рассчитайте напряжения E 1 , E 2 и E 3 .
      3. Общий ток на два параллельно соединенных резистора составляет 3 А. Сопротивление R 1 составляет 10 Ом, а сопротивление R 2 составляет 40 Ом. Используя формулу делителя тока, рассчитайте токи I 1 и I 2 .
      4. Резистор 5 Ом, R 1 , и резистор 20 Ом, R 2 , подключены параллельно друг другу и последовательно с резистором 6 Ом, R 3 .Вычислите полное сопротивление этой последовательно-параллельной цепи.
      5. Вам даны три резистора по 100 Ом, которые нужно соединить вместе. Опишите три возможные конфигурации цепей и вычислите их значения общего сопротивления.

      Контрольные вопросы — ответы

      1. (а) 115 Ом, (б) 66,7 Ом, (в) 10200 Ом, (г) 9,74 Ом, (д) ​​5700 Ом, (е) 8,57 Ом
      2. E 1 = 100 В, E 2 = 60 В, E 3 = 40 В
      3. I 1 = 2.

    Добавить комментарий

    Ваш адрес email не будет опубликован. Обязательные поля помечены *